Этого треда уже нет.
Это копия, сохраненная 18 января 2018 года.

Скачать тред: только с превью, с превью и прикрепленными файлами.
Второй вариант может долго скачиваться. Файлы будут только в живых или недавно утонувших тредах. Подробнее

Если вам полезен архив М.Двача, пожертвуйте на оплату сервера.
 .jpg346 Кб, 1600x1000
Глупых вопросов тред Защитник !HOHLOVjqt6 420851 В конец треда | Веб
Здесь ты можешь написать глупый вопрос, но не глупый ответ.
2 420853
>>0851 (OP)
Смотри, анон.
1. Земля движется вокруг Солнца.
2. Следовательно обладает кинетической энергией, пропорциональной квадрату скорости.
3. Проведем мысленный эксперимент — удалим из вселенной все объекты кроме Земли и Солнца.
4. Теперь невозможно определить, что вокруг чего вращается (вроде бы)
5. Вопрос: куда делать кинетическая энергия Земли? Не нарушается ли закон сохранения энергии?
Orbit2[1].gif26 Кб, 200x200
3 420856
>>0853
Задача двух тел
4 420858
>>0856
Но тут три тела! Есть ещё и крестик! Шах и мат!
5 420859
>>0853
Уровень кинетической энергии действительно зависит от системы отсчёта. Это относительная величина. Разумеется, если удалить все тела и точки отсчёта, она будет неопределимой.
6 420862
>>0858
Каспаров, ты что-ли? Центр масс не тело!
7 420865
>>0853
Неверно,
1. При движении по орбите тело испытывает ускорение, то есть действует сила =>
2. Система отсчета получается не инерциальная =>
3. Относительность движения отсутствует. Чётко определяется кто вокруг кого движется.
8 420867
>>0865

>тело испытывает ускорение


ну так и что епта, любое тело будет испытывать ускорение при применении к нему силы, но от этого относительность не пропадает.

> то есть действует сила =>


в том-то и вопрос, почему для этой системы 3-й закон Ньютона внезапно перестал работать, хоть система и простейшая - ни искривления пространства, ни квантовых эффектов и подобной хуйни.
9 420868
>>0867

>3-й закон Ньютона внезапно перестал работать


Третий закон Ньютона сформулирован только для инерциальных систем отсчёта. Чтобы он работал в неинерциальной, в него нужно внести поправки, зависящие от степени и характера отклонения данной системы от инерциальной.

>но от этого относительность не пропадает


В инерциальных системах отсчёта ускорение величина абсолютная, если мы говорим про классическую механику. В неинерциальной системе отсчёта, связанной с Землёй, будут существовать весьма таинственные силы, которые приводят к эффекту Кориолиса, влияют на маятник Фуко, заставляют весь небесный свод - в частности, Солнце - вращаться вокруг земли с охуительной скоростью.
10 420869
>>0868

>Чтобы он работал в неинерциальной, в него нужно внести поправки, зависящие от степени и характера отклонения данной системы от инерциальной.


ок, с поправками можно представить вращающийся объект как неподвижный?

>будут существовать весьма таинственные силы, которые приводят к эффекту Кориолиса, влияют на маятник Фуко, заставляют весь небесный свод - в частности, Солнце - вращаться вокруг земли с охуительной скоростью.


а их можно будет представить как-то нормально, ну чтобы по ним прийти к вращающему объекту и определить эти таинственные силы как его влияние?
11 420870
Как сумели подсчитать сотни тысяч видом одного насекомого не повторяясь и отличая каждый?
12 420871
>>0869

>ок, с поправками можно представить вращающийся объект как неподвижный?


Нет. Мы можем модифицировать закон, чтобы расчёты согласовывались с действительностью, но инерциальной система от этого не станет.

>а их можно будет представить как-то нормально, ну чтобы по ним прийти к вращающему объекту и определить эти таинственные силы как его влияние?


Да, можно. Понимаешь, в неинерциальных системах отсчёта нарушается один очень важный физический принцип, а именно - изотропность пространства. Т.е. в пространстве инерциальной системы отсчёта не существует "выделенных направлений", вдоль которых действуют силы, необъяснимые с точки зрения взаимодействий между телами. А в неинерциальных системах отсчёта такие силы, возникающие словно бы из ниоткуда, живут и здравствуют, и нарушают третий закон Ньютона.
13 420872
>>0870

>не повторяясь и отличая каждый


Между прочим, в систематике такое встречалось довольно часто - внешние признаки могли бы быть очень схожими, поэтому несколько видов ничтоже сумнящеся объединяли в один. Сейчас существуют несколько классификаций, основывающиеся на разных принципах.
Отнесение той или иной популяции к какому либо виду сейчас вычисляется. Существуют формулы, которые по внешним, физиологическим и генетическим признакам оценивают степень родства между двумя популяциями. Если она высока - перед нами с высокой вероятностью один и тот же вид.
14 420873
>>0872
Нашли 800 видов, например, как они собираются определять новый ли у них вид или это уже найденный? Заново каждый раз сверять со всеми 800?
15 420874
>>0871

>возникающие словно бы из ниоткуда


вот тут проблема, неужели нельзя их определить? Ну что-то вроде "пространство ускоряется т.к. опирается на вращающее тело" или подобное?
16 420875
>>0873

>Нашли 800 видов


Это не так работает. Скорее "нашли 800 явно различных между собой организмов". И да, приходиться пересчитывать. Впрочем, пересчитывают тоже не всегда и не всех. Если различия фундаментальны - скажем, в анатомии, или в физиологии, - даже одного - двух факторов может быть достаточно при условии, что подобные факторы достаточно распространены - т.е. нельзя просто так взять и сказать, что это - аномалия развития.
17 420876
>>0874
Их можно определить. Самый наглядный пример - маятник Фуко, сила Кориолиса, неравномерное размывание берегов рек в разных полушариях. Если взять закон всемирного тяготения, можно было бы и здесь усмотреть странность: как такая маленькая, нетяжёленькая Земля, от которой даже Луна норовит съебать, может заставить миллиарды тяжелейших звёзд вращаться вокруг ней самой?
18 420879
>>0876

>маятник Фуко, сила Кориолиса, неравномерное размывание берегов рек в разных полушариях.


тут все равно при рассчетах пользуются системой сточкой отсчета на Солнце.
А вообще, если сам факт вращения себя относительно чего-то дает такие эффекты, получается так можно до бесконечности определять "странные силы": вращение вокруг Солнца, галактики, центра Вселенной, вращение самой Вселенной вокруг чего-то и наконец вращение самого пространства как его неравномерность.
19 420881
>>0875
Муравьёв в мире более 14 000 видов. Как их пересчитать, чтобы определить новый вид? А вдруг это старый, но который просто забыли при таком количестве? Или новый, н оего принимают за один из открытых?
20 420893
>>0881
По морфологии обычно делают.
21 420911
>>0881
Сейчас есть компьютеры. Свойства живого организма формализуют и прогоняют по базе данных.
22 420926
Как понять что такое фотоны, нейтроны, атомы, почему они есть и имеют определённые свойства и почему существует жизнь?
23 420941
>>0926
Открываешь учебник физики. Читаешь.
Я ответил на твой вопрос?
24 420960
>>0941
Умный дядя, открой его сам, а потом уже говори. Да и это глупых вопросов тред.
25 420961
>>0926
Гуглишь определения, и если не понятно - гуглишь каждое слово из этих определений.
26 420966
>>0851 (OP)
как отражается свет? на физическом уровне. есть маленький фотон есть столь же маленький атом (из них состоит зеркало или любпя поверхность ок). тем не менее фотон как то попадает и почему то меняет направление. как это происходит?
27 420971
Смотрел тут один фильм, где люди в Австралии ждали примерно 12 часов, когда до них дойдет здоровенная взрывная волна от удара астероида в Атлантический Океан. И стало интересно, а сколько бы это заняло времени ИРЛ? Это было бы со скоростью звука или как?
image006[1].jpg17 Кб, 592x238
28 420972
>>0966
Гугли когерентное и некогерентное рассеяние. Пикрелейтед.
20110311Houshu.ogv.480p[1].webm7,3 Мб, webm,
508x480, 1:00
29 420973
>>0971
В открытом океане волны цунами распространяются со скоростью √(g⋅H), где g — ускорение свободного падения,
а H — глубина океана (так называемое приближение мелкой воды, когда длина волны существенно больше глубины).
При средней глубине 4 км скорость распространения получается 200 м/с или 720 км/ч.
Вебм-релейтед.
31 420977
>>0961
Но мне всё равно непонятно почему они именно в таком виде существуют с характеристиками. Почему частица может не иметь массы, почему гравитацией обладают все предметы, если по законам физики энергия не берётся из ниоткуда. Что будет, если бесконечно расщеплять атом.
32 420980
>>0977

>Но мне всё равно непонятно почему они именно в таком виде существуют с характеристиками.


На этот вопрос на данный момент однозначного ответа нет. Есть только разной степени адекватности теории. К тому же рассчитано, что если бы какие-либо хотя-бы чуть-чуть отличались, то вселенную какую мы наблюдаем не было бы, и все фундаментальные взаимодействия шли бы по другому.
33 420981
>>0977
Как писали, есть так называемая проблема тонкой настройки, т.е. один коэффициент, например, в 1000000000 раз больше другого. Почему -не ясно. Например, так обстоит дело с массами частиц: почему одни элементарные частицы намного тяжелее других? Если бы это соотношение масс было бы другим, то и соотвношение газов (водород, гелий и т.д.), из которых создавалась бы ранняя вселенная, тоже отличался. Далее, как понять, из чего сделана вся материя? Ответ простой, если не переходить к деталям. Предстаим дом, в нем 2 типа кирпичей: кварки (которые могут быть связаны очень крепко - сильное + электромагнитное взаимодействие) и лептоны (которые связаны не так крепко - слабое + электромагнитное). И те и другие имеют 3 поколения, которые отличаются друг от друга массой. Но просто кирпичей мало - нужно связывающее вещество - калибровочные бозоны (фотоны(электромагнитное взаим-вие), глюоны (сильное), W/Z бозоны (слабое).
34 420982
>>0977

>Но мне всё равно непонятно почему они именно в таком виде существуют с характеристиками.


Какие характеристики ты имеешь в виду?Загугли каждую конкретную характеристику у конкретной частицы, и вникни в её природу.

>Почему частица может не иметь массы, почему гравитацией обладают все предметы, если по законам физики энергия не берётся из ниоткуда.


Связь энергии и массы ещё давно сформулировал Альберт Эйшнтейн, выводя формулу E=mc2
Что касается природы инертной массы у частиц - это обсуждалось тут: >>420173

>Что будет, если бесконечно расщеплять атом.


Ну, по идее он расчепится на ядра гелия4, которые трудно ращепить, либо же, при высоких энергиях распадётся на нуклоны.
Нуклоны же ещё труднее расщепить, потому что между кварками - сильное взаимодействие и конфайнмент.
18-2.jpg32 Кб, 339x244
35 420987
Сап, аноны.
Начался курс сопромата, там в начале семестра определение внутренних силовых факторов при осевых нагрузках, в связи с чем возникает вопрос о природе возникновения сил упругости и строения кристаллической решетки металла:

Во-первых, каким вообще образом решетка существует? Что мешает положительным ионам металла присоединить свободно "плавающие" в металле электроны? Каким образом электронный газ в металле компенсирует силы отталкивания между положительными ионами? Как эта система сохраняет устойчивость (ведь вряд ли обеспечвается постоянная по объёму концентрация электронов или ионов, следовательно не во всех точках объёма кулоновские силы скомпенсированы)? И ещё: ну допустим, мы сжали стержень, расстояние между ионами металла сократилось, и кулоновская сила возросла. Получается, это изменение в силе отталкивания и будет упругой силой, уравновешивающей внешнюю нагрузку? Но ведь сила отталкивания пропорциональна (1/r^2), в то время как сила упругости по з-ну Гука пропорциональна (r), где r - линейное расстояние вдоль оси стержня. Отсюда получается, что сила отталкивания по мере сжатия стержня будет расти быстрее чем сила упругости, значит вопрос о микроскопической природе упругих сил остаётся открытым.

Понимаю, что в пределах сопромата это всё не важно, но всё таки хочется ликвидировать недопонимание. Помогите, пожалуйста.
36 421001
>>0851 (OP)
почаны каталог разшырения не видит папку Scihub
37 421002
>>1001
подробнее
38 421010
>>0987
Тут надо понимать несколько вещей: первое - закон Гука приближенный и работает для неслишком сильных деформаций, второе - всё таки на межатомном уровне начинается игра двух взаимодействий - электромагнитного и сильного.
39 421012
>>1010

> и сильного.


может таки слабого?
40 421013
напомните название протеина который может самовоспроизводиться и он вреден, смертельно или нет уже не помню.
было популярно некоторое рвемя назад.
он так-же термически стоикий.
sage 41 421014
>>1013
прион. вспомнил.
Осозанные сновидения AllaYaVBar 42 421020
Что вообще из себя представляет осознанный сон и как в него попасть?
43 421023
>>1012
сильного
44 421027
>>1023
Ща бы хуйню спиздануть на двачах... Какое нахуй сильное взаимодействие в законе Гука? Ты ебу дал. Там только кулоновские силы.
баунти.jpg18 Кб, 480x199
45 421030
>>1020
Сон это такая хуйня связанная с воображением и памятью, но в отличии от бодрствования в ней отключены некоторые функции вроде логики. ОС проявляются только во время пробуждения, и по сути просто включается рефлексия. хотя собственно не факт это может быть просто навязчивое желание попасть в определённое место в данном случае сон. И по сути это, сон о том, что ты попал в сон.
gregariousquarks738 (1).jpg71 Кб, 738x224
46 421038
>>1027

>Какое нахуй сильное взаимодействие в законе Гука?


Ты только посмотри на эти "пружинки" в адронах при кварковом конфайнменте.
47 421057
>>1038
Ты типа в тонкоту пытаешься, ну ок. Подыграю.
48 421062
>>1038
проорал, спасибо
49 421085
>>1014

>прион


>Вопрос о том, считать ли прионы формой жизни, в настоящий момент является открытым.


Так они живые или нет, что думает анон?
Больше похоже на самовоспроизводящиеся машины, самореплицирующиеся машины Фон Неймана и серую слизь.
50 421090
Подрочил я значит в душе, слив нормальный, вся сперма съебала в канализацию, но что если немного осталось. Может ли она как нибудь в последующих использованиях душа прицепится к ногам? Забеременить так можно?. И еще вопрос по данной тематике: если обдроченные трусы закинуть в стиралку, то останутся пятна на другой одежде? Знаю, вопросы сверх тупые, но все же,
успокойте меня
son[1].jpg52 Кб, 800x533
51 421092
>>1020
Осознанное сновидение (англ. lucid dreaming) — изменённое состояние сознания,
при котором человек осознаёт, что видит сон, и может в той или иной мере управлять его содержанием.
Это некое пограничное состояние между фазой быстрого сна и бодрствованием.
Осознанные сновидения являются предметом научных исследований, их существование подтверждено, в частности, японскими учёными.
Я видел у Митио Каку, вроде-бы мозгопочту и почту сновидений: https://www.youtube.com/watch?v=cO6njjrRuPg

В процессе осознанного сновидения возможно моделирование любых ситуаций, невозможных в обычной жизни,
что открывает широкий спектр возможностей для прикладного применения:
Виртуальная реальность. Осознанный сон можно рассматривать как виртуальную реальность,
где практически отсутствуют какие-либо ограничения.
Психотерапевтический аспект. Научившись уверенному управлению действиями и событиями
в осознанном сновидении, например: полётам во сне вместо падений, изменению сюжетных линий,
схожих, типовых и повторяющихся снов сновидец имеет возможность испытать и перенести ощущение
уверенности в свою повседневную жизнь и, в том числе, избавиться от некоторых страхов или фобий.

Да, это всё моделируется, и да это можно эмулировать на принципах ИИ.
Ведь интеллект - процесс расширения частной меры, и он может быть направлен во времени - наилучшим для тебя образом.

Так как сон действительно связан воображением и памятью, как написал этот: >>1030
это значит, что тебе действительно предстоит пережить то, во что тебе хотелось бы вернуться во сне, и ещё и правильно.
Тогда и получится. Но для этого надо бы забыть суть происходившего, и таки увидеть что-то новое для себя в момент воспоминания,
или ощутить иллюзию порождения этого как-бы в эти конкретные моменты.
Для этого должна быть тонкая настройка нейросети, возможно даже ИНС - мемристорной или фотонной.
52 421093
>>1090

> если обдроченные трусы закинуть в стиралку, то останутся пятна на другой одежде?


нет
11b2986261e3[1].jpg87 Кб, 600x458
53 421094
>>1090
После периода созревания, составляющего около 64 дней, сперматозоид может сохраняться в организме мужчины до месяца.
В эякуляте они способны выжить в зависимости от условий среды (свет, температура, влажность) до 24 часов.
Во влагалище сперматозоиды погибают в течение нескольких часов.
В шейке матки, матке и фаллопиевых трубах сперматозоиды остаются живыми до 3 суток.

Короче, залей всё это дело кипятком, и трусы в кастрюлю себе кинь.
54 421095
>>1093
Почему?
55 421096
>>1094
хуйню какую-то написал
56 421098
>>1093
всё смоет
алсо если правильно и вовремя втереть не останется и без стиралки даже
58 421100
>>1099
ок, а забеременеть из-за пролитой кончи в душе это все таки детские выдумки?
59 421102
>>1100
нет, ну если быстро зайти подобрать большой кусок с пола и пропустить через все овальные губы то появится очень маленький шанс
так что если после тебя будут заходить любители собирать малафью с пола и засовывать её оче глубоко - то стоит начинать бояться, лул
60 421103
>>1102
Фух, спасибо успокоил, теперь буду спокойно дрочить свой так сказать пэнис
Вопрос 61 421104
Курица или яйцо ?
62 421105
>>1104
тебе решать
64 421113
>>1104
ИМХО, яйцо одного из этих, но чуток повыше, из которого мутировала - таки курица:
Домен: Эукариоты
Царство: Животные
Тип: Хордовые
Класс: Птицы
Отряд: Курообразные
Семейство: Фазановые
Род: Джунглевые куры
Вид: Банкивская джунглевая курица
Форма: Курица
65 421125
>>0853
Эйнштейн рассматривал это в ОТО. Коротко: масса и инерция существует только в связи со всей остальной вселенной. Без вселенной нету ни того ни другого.
7278342.jpg62 Кб, 669x381
66 421131
привет любителям науки и около того.
публикую сей вопрос тут, ибо некогда разбираться, где он тут уместен.

допустим (!) на минутку, что есть некая тусовка (рептилойдов, лол) людей, которые заведуют и по факту владеют всеми и вся.
почему они, если, опять же таки, допустить, вместо повышения уровня образования, культуры и, в последствие, налогов, либо смотрят на этот муравейник безразлично, либо способствуют разжиганию войн, которые не особо-то себя и оправдывают в качестве методов борьбы с перенаселением?
пример - Европа. высокий (хваленый) уровень культуры и образования населения, высокие налоги... и низкий уровень рождаемости.
вложитесь повсюду в такое и через век, по идее (или 2, думаю, максимум, века) планету будет не узнать, население пойдет на спад.

при чем тут налоги: антипример - СССР. высокий (относительно) уровень образования и культуры (отметем тут учет того, что насленеие двигалось и развивалось туда, куда и в каком ключе направляла партия. возьмем только некую культурность и образованность).
добавляем налоги, постепенно, и вуаля.
67 421134
>>1131

>методов борьбы с перенаселением


Щито, блеать? Какое перенаселение?

Условно, если бы Кипр был плоским, то на его территории вполне уместилось бы человечество.
https://thequestion.ru/questions/1548/esli-vsekh-lyudei-zemli-sobrat-v-odnoi-tochke-to-kakuyu-velichinu-ploshadi-sostavit-eto-prostranstvo
68 421142
>>1134
То, в которое верит большая часть современной западной илитки и весь ООН.
69 421145
>>1142
Ну и долбоебизм.
70 421151
>>1131
Белое население Европы имеет положительный прирост. Пруфы в гугле на первой же странице.
71 421154
>>1134

>Щито, блеать? Какое перенаселение?


>Условно, если бы Кипр был плоским, то на его территории вполне уместилось бы человечество.


А если в кубический коммиблок запихивать, то и одного кубического километра хватит.
А если спрессовать и ноги всем отрезать, то и в половину кубокилометра поместятся.
Действительно, какое перенаселение? Нужно больше биомассы. Уровень жизни не нужен. Дал бог зайку, даст и лужайку камеру в крольчатнике.
72 421157
Дано:
куб со стороной a
куб имеет массу m
на куб действует сила тяжести g
куб имеет коэффициент трения с поверхностью мю
куб имеет начальную скоростью V, движется по горизонтальной опорной поверхности
куб имеет плотность ро
куб имеет удельную теплоёмкость c
куб имеет теплопроводность лямбда
до какой температуры нагреется опорная поверхность куб на момент его остановки?
73 421158
>>1092
У меня были такие случаи. Ну, просто во сне понимаю, ёпт, да я же сплю, это всё ненастоящее и просыпаюсь.
74 421159
Реально ли записывать в себя тексты с книг и воспроизводить их на бумагу во время экзамена? Почему сраная книга или любой другой носитель так легко умещают столько инфы и так легко её дают считать человеку, а человеческий мозг с трудом запоминает всё это говно?
75 421160
>>1030
Какова вероятность, что на заводе изготовителя шоколадки с твоей пикчи по ошибки захуярят три кусочка?
76 421161
>>0982

>Связь энергии и массы ещё давно сформулировал Альберт Эйшнтейн, выводя формулу E=mc2


Но это же всё равно что про магию читать, каждый предмет взаимопритягивается с другим, являясь неиссякаемым источником энергии. Как масса может порождать энергию сама по себе? И как гравитация может искажать время, замедляя его? На планетах возле чёрной дыры можно прожить тысячи лет за годы?

>Нуклоны же ещё труднее расщепить, потому что между кварками - сильное взаимодействие и конфайнмент.


Что останется, если продолжать расщепление?
77 421164
>>1157
В твоем случае с параметрами, которые ты описал - нагреется весь куб сразу, так как школьная модель теплопроводности подразумевает мгновенную передачу тепла. Строго, нужно учитывать характерную кристалическую структуру куба, если ты хочешь строгий ответ.
Приблизмтельную оценку думаю можно провести, зная скорость звука еще.
78 421165
>>1161
Да для тебя и инструкция по сборке стула от икеа магией будет.
79 421166
>>1164
Тогда ещё такой вопрос, есть энергия, есть теплоёмкость и масса. Если так поделить, то мы узнаем, на сколько бы нагрелось всё тело. Но если мы узнаем точный ответ, то ведь явно, что в пятне контакта температура будет больше, чем та, если весь куб нагреть этой же энергией?
80 421167
>>1161

> Как масса может порождать энергию сама по себе?


представь волну на море. Волна на воде- это искажение двумерного пространства в трехмерном (не придирайся сейчас, я условно так обозначил, для лучшего представления).
Масса волны тем больше, чем больше ее энергия. А вещество в нашем мире можно представить как интерференцию волн в очень многомерном пространстве.
Отсюда становится понятна связь массы и энергии.
Если волну на море резко остановить, ее энергия перейдет в другую форму. То есть волна
условно "выделит" энергию и исчезнет.
81 421171
Просто киньте формулу нагрева от трения. Поделить кинетическую энергию на время от начала потери кинетической энергии до полной остановки и получить ватты, которые будут греть поверхность - это правильный ход?
82 421172
>>1085
Ну а что такое жизнь вообще? Вирусы можно считать живыми? Если организм может в саморепликацию, то почему его нельзя считать живым?
83 421173
>>1161

>Что останется, если продолжать расщепление?


Энергии, содержащейся в глюонной струне, сзывающей кварки внутри адрона, хватит, чтобы образовать по кварку на концах разорванной струны, и тем самым из одного адрона ты получишь 2 адрона, если на пальцах.
Это конечно если не брать в рассмотрение экстремальных состояний вроде кварк-глюонной плазмы.
84 421178
>>1161

>Как масса может порождать энергию сама по себе?


Масса не порождает энергию сама по себе. Энергия это физическая характеристика какого-либо физического объекта (такая же как скорость и импульс) и без этого объекта она не существует. Формула Эйнштейна просто связывает эти два понятия. Кроме того, не стоит забывать, что мс2 это энергия покоя, а не полная энергия объекта.
Когда, например, говорят, что в результате реакции выделилось столько-то энергии имеют ввиду, что в результате реакции образовались какие-то элементарные частицы с такой-то суммарной энергией.
85 421189
В каком году было обнаружен химический и относительный состав воздуха?
86 421190
>>1173

>на концах разорванной струны

1489739183022.png19 Кб, 500x590
87 421194
Глупый вопрос:

Какими полезными научными исследованиями и в какой сфере можно заниматься абсолютно без лаборатории (то есть есть при помощи одних только мозга и комплюктера) и с низким порогом входа по тому что ты должен перед этим выучить (например если надо учится перед этим около года с нуля но не пять)?

Дополнительный вопрос: если бы вы хотели запилить краудсорсную платформу которая сначала обучает индивида всему что ему надо знать для произведения исследований а потом дает ему возможность их проводить, реалистично, на какую из проблем науки вы бы напустили 10-100 к мимокроков?
88 421195
>>1194
Теоретические исследования в любой области.
fry.jpg7 Кб, 259x194
89 421197
>>1195
С одной стороны несомненно да, но с другой стороны насколько я понимаю после года занятий вряд ли возможно внести какой либо вклад в теоретическую математика или физику, это кажется мне почти невозможным. В то же время я уверен что должны быть хоть какие ни будь области науки в которых одна из главных проблем это непосредственно нехватка рук.

У людей есть большой интерес к науке. Запилили один веб-сайт где просили людей смотреть на фотографии и если они замечали там фотографии яиц червей, их отмечать(это нужно было для кого-то исследования). Еси я не ошибаюсь больше 10000 людей этим занимались просто так. Так вот мой вопрос какую самую полезную краудсорсную платформу можно сделать чтобы двигать науку вперед.
90 421201
>>0851 (OP)
Какова природа математической истины? Вот 1+1=2 это вроде как истина, но истина не такая как "Расстояние от моей лобковой кости до конца головки моего члена в эрегированном стоянии составляет 13см, что является средним и нормальным значением". Знаю людей утверждающих что математика это вообще не про истину и не про объективность, а просто маняфантазёрство.
91 421207
с компьютерами и интеллектуальной работый люди все меньше взаимодествуют с физическим-асязаемым миром.
к примеру, труд программиста или журналиста, не обладает массой или структурой которую можно потрогать, у него нет запаха. они не переводят свои мысли в физические объекты.

хотел бы узнать, как это влият на нашу жизнь? пагубно или нет?

я помню фото Паулинга -- химик, он конструировал молекулы из сфер, как конструктор. не знаю как это повлияло на его открытия, и повлияло ли, но что если это может помочь в решении тех или иных задач?
92 421208
>>1207
известно, что неподвижный образ жизни, вреден.
но меня интересует не только это. а связь, если есть, с воплащением наших идей в осязаемых-объектах.
одно дело посмотреть картинки природы и совсем другое, погулять в этом месте.
некоторые люди предпочитают бумагу. можно читать и с экрана, но им хочется бумагу.

эксперимент лучше запоминается если бабахнет, завоняет, так? чем если прочитать об этом в книге.
93 421209
>>1201

>математика это вообще не про истину и не про объективность, а просто маняфантазёрство


Так и есть. На западе ее вообще к гуманитарным относят.
94 421212
Вечного движка не бывает.
Но вот смотрите. Если к примеру взять материю отрицательной массы. Она несомненно будет бегать от обычной материи, тем самым двигаясь с постоянным ускорением в каком либо направлении. Энергия от движения перерабатывается в супер генераторе и брав эту энергию образовывать всё новые и новые пары массовой и антимассовой частицы.
В чём подводные камни?
95 421213
>>1212
Одинаково заряженные частицы тоже друг от друга отталкиваются, и что?

> Энергия от движения перерабатывается в супер генераторе и брав эту энергию образовывать всё новые и новые пары массовой и антимассовой частицы


КПД будет <100% всё равно
96 421215
>>1213
А частица с массой притягивается к частице с отрицательной массой, которая от массы убегает.
97 421217
>>1212
Так это использование энергии гравитационного взаимодействия. Да, такой двигатель будет вечным, в плане работы, но не будет "Вечным двигателем" - энергия не будет браться из ниоткуда.
Да и как писали выше - КПД <100% всё равно будет
98 421218
>>1197

>В то же время я уверен что должны быть хоть какие ни будь области науки в которых одна из главных проблем это непосредственно нехватка рук.


История, антропология, палеоантология.
Короче всё, что так или иначе связано с копанием в земле или архивах: нет долгого специализированного обучения (на месте объяснят, как надо и отпиздят, если что-то неправильно сделаешь, тоже на месте) мало волонтеров, мало денег и в то-же время выше нулевого шанс сделать какое-нибудь ебически крутое открытие. Homo Naledi из недавних, например, или Троя из эпических
temp1.png13 Кб, 620x578
99 421220
Вот есть квантовая запутанность, там еще про парадокс какой-то кукарекают, а я вот нихуя не пойму. Возмем йобу пикрил (естественно она непрозрачна), кинем два шарика, встряхнем, и вставим заслонки. Далее распиливаем йобу пополам. Чем отличается от этих ваших фотонов? Мы тоже не знаем, в какой половинке че, но ведь шары нихуя не запутаны, открытие одной из них тоже покажет, какой шар во второй, хоть на альдебаране, но он же на неё никак не влияет.
100 421221
>>1220
И блядь какая нахуй неопределенность? Тобишь частица имеет оба состояния, а когда измеряешь выясняется одно конкретное? Ну так давай колдун проклятый, заставь частицу взоиметь определенное состояние, нихуя? Где пруф, что это состояние не было определено изначально? Наебую вас на доллары, а вы ведетесь.
101 421222
>>1220
двачую вопрос, где доказательства что частицы до измерения их вообще находятся в суперпозиции, ведь на них по условию еще никто не смотрел.
102 421223
В атомных часах n-знаков после запятой. Верно ли, что время иррационально, поскольку знаков может быть бесконечно? Тогда как эта бесконечность может увеличиваться, сбрасываться до нуля и так непрерывно?
103 421226
Господа, позвольте поинтересоваться.

Я тут читал ко кварках и бозонах и понял мало.
Можно обяснить попроще или в сравнении:

Можно ли "увидеть" атом или кварк или протон? Или это как плазма типа кусочек огня? или это пустота вообще даже при биллионном увеличении?
Потом, выходит что кварки это вечный двигатель, так? Откуда у них энергия? От бозона Хигса пульсировать бесконечно?
104 421227
Почему когда летали на луну сначала выходили на орбиту Земли и только потом совершали манёвр для полёта на луну? Ведь им же пришлось бы гораздо меньше топлива потратить, если бы они сразу на луну летели?
1043089.jpg158 Кб, 592x864
105 421228
Почему не могут решить проблему трех тел, те вывести формулу, по которой можно точно узнать, где будут тела через х времени? Ведь при одинаковых исходных результат может быть лишь единственный?
106 421230
>>1226
Электронные микроскопы могут видеть отдельные атомы
https://habrahabr.ru/company/ibm/blog/151574/
107 421231
>>1222

> где доказательства


Неравенства Белла
Three-bodyProblemAnimationwithCOM.gif501 Кб, 300x300
108 421232
>>1228
Как я понял из вики, точную формулу вывести невозможно:
Точное решение
Брунс и Пуанкаре доказали, что систему дифференциальных уравнений для движения трёх тел невозможно свести к интегрируемой, разложив её на независимые уравнения. Открытие показало, что динамические системы не изоморфны. Простые интегрируемые системы допускают разложение на невзаимодействующие подсистемы, но в общем случае исключить взаимодействия невозможно
109 421246
>>1227
Даже интересно стало, как по-твоему они летали, и как это "сразу на луну".
110 421247
>>1232
Охуенно, просто скопировал из вики. Я про то, что есть строгая же зависимость: начальные параметры>время>текущее положение.
111 421248
>>1247
Зависимость есть, а формулы нет, прикинь. Ждет когда ты ее выведешь.
s69-34071.jpg75 Кб, 640x502
112 421249
>>1227

>им же пришлось бы гораздо меньше топлива потратить


Пришлось бы больше, кроме того нужно было провести тесты систем в космосе, вытащить Водолея из-под Одиссея и изменить плоскость орбиты.
113 421251
>>1248
А хуле мне её выводить? Они же умные, пусть и выводят. Придумали 3 уравнения, каждое от других зависимо, что бы отъебались. Пусть нормально решают, за что им государство зарплаты платит?
114 421255
>>1231
Можно вкратце? Как я понял ребята измеряли состояние 2 фотонов, но при этом их соответственно меняли. Как же они выяснили что фотоны до измерений были в другом состоянии?.
115 421256
>>0851 (OP)
А как работает всякий контрастный душ и утренний бег? Особенно контрастный душ, я чёт ума не приложу какой там механизм этой закалки и как нагрузки могут улучшить здоровье на длинной дистанции.
116 421261
>>1255
не ебу за фотоны и хуёны но у тебя просто по условию поулчается что раз во время измерения состояние меняется то до него оно было иное, чисто лингвистически.
117 421262
>>1261
так и я о том же, просто кто-то решил что измерение меняет состояние, но само условие подобной теории по сути недоказуемо и попахивает солипсизмом. Все равно что сказать "сзади тебя стоит белый клоун который исчезает при его детекции(взгляде, осязании, попытке ткнуть палкой и т.д.)".
118 421263
>>1255
Элементарно, Ватсон.
Например.
1.Измерили импульс, записали,
2.Измерили координаты, записали.
3. Снова измерили импульс, записали и обнаружили, что он не совпадает с импульсом из пункта 1. Причем если последовательно измерять импульс без измерения координат, то он остается одинаковый.
1408626553detenysh-vombata-4.jpg64 Кб, 650x365
119 421264
>>1262
ебать даун. Гугли модельно зависимы реализм. На твои филосовские психозы людям решающим реальные проблемы просто похуй, до тех пор пока модель даёт самые точны предсказания то пользуемся ей. Хочешь выебнуться какой ты ебаца разгрызатель истины то запили модель которая даст лучшие предсказания нежели имеющаяся. У тебя вся жизнь, время пошло.
120 421265
>>1263

>Измерили координаты


эм, как? "коснувшись" фотона? А не то же самое ли это что измерять импульс? Ну с точки зрения фотона а не результатов измерения. допустим они повлияли на фотом измерением, но откуда они знают что предыдущее состояние именно суперпозиция а не, скажем, просто противоположное или вообще такое же а менялся иной параметр(например, энергия)?
121 421266
>>1264
ты тредом ошибся, уебок, тут отвечают на глупые вопросы а не гуглят. Почитай тред еще раз, если бы все гуглили перед тем как сюда писать, тред был бы пустой.
122 421267
>>1266
Глупые но не тупые, деб.
123 421268
>>1265
потому что результат следующее измерение не удаётся предсказать используя данные только что сделанного.
124 421269
>>1268
э, и че? Первое измерение то было не поляризации а чего-то там еще, как ты можешь сравнить два различных измерения?
125 421270
>>1269
С его это они блять различные всё что их отличает это время, и за это время согласно классической физике интересующий нас объект должен был бы оказаться вот тут, Но чёто его нет..
126 421271
>>1270
точнее он есть но не всегда.
127 421272
>>1270
я о том что ты сначала меряешь координаты, затем импульс, это же разные измерения(измерения разных величин).
128 421274
>>1272
Так измерь всё сразу... кто не даёт?
129 421275
>>1274
если измеришь все сразу, GOTO к первому вопросу: откуда ты знаешь что было до измерения и как именно измерения повлияло, и повлияло ли вообще. Опиши еще раз эксперимент, последовательно.
130 421276
>>1275
Как кому еще первому вопросу? Какой еще до измерения? Ты измеряешь импульс и координату, заносишь в свои уравнения и на выходе получаешь нихуя, в какой то момент придётся решить что говно - твои измерения или твои уравнения. Что решили физики мы прекрасно знаем.
131 421277
>>1275
>>1276
Чет сижу угораю с вас, спорят про квантмех СЛОВАМИ.
132 421278
>>1276
по вот этому:
>>1255

что ж, ере раз повторю его: с чего вы взяли что до измерений было другое состояние? Особенно поляризация.
133 421279
>>1278
>>1276
>>1272
>>1270
>>1269

у меня возникает ощущение, будто вы думаете, что состояние было, и вы его измерили. пока фотон вас не трогал, у него нет конкретного состояния.
Ozelenenie-balkona-dikim-vinogradom[1].jpg84 Кб, 500x375
134 421280
>>0851 (OP)
Аноны, назовите самый энергоэффективный продукт,
с помощью которого следовало бы быстро и легко превращать энергию солнца
в химическую энергию питательных веществ для организма.
Думал изначально выращивать у себя виноград, но чё-то там воды дохуя и он быстро портится.
А надо чё-то такое, что хранилось бы веками.
135 421281
>>1280
Так изюм же.

А вообще - бобовые всякие, типа сои, например. Но они как на пике расти не будут
136 421282
>>1280
Стабильное состояние энергоэффективного фотосинтеза - C4-путь, к которому возвращались немало организмов:
https://lenta.ru/news/2012/12/26/c4/
Фотосинтез по С4 пути проводят многие культурные растения - кукуруза, сахарный тростник и сорго.
Но лучше поставить солнечный элемент и ультрафиолетовую лампу,
потому что электрохимического синтеза из углеводов - я не нашёл.
Может кто подскажет чё-нить лучше, буду рад, самому интересно...
137 421283
>>1281

>Так изюм же.


Да, анон, годно. Изюм - это сушёный виноград. Я правда не знаю как его делают,
но не думаю что он может пролежать не портясь - веками.

>А вообще - бобовые всякие, типа сои, например.


У этих год всего.
>>1282

>кукуруза, сахарный тростник


Вот из этого можно выкристаллизировать сахарозу, а она уже может и веками храниться в сухом темном месте.
138 421284
>>1279
ну так это из определения квантовой сцепленности - два фотона связаны состояниями инверсно, и при переходе из суперпозиции в одно, другой переходит в другое. Переход происходит при попытке измерения состояния, соответственно узнать что там была суперпозиция а не определенное состояние - нельзя. Что приводит к основному вопросу - а с чего вы взяли что там суперпозиция? Логика говорит нам что там было то состояние что мы измерили, и оно было при рождении пары а не при измерении.
139 421285
>>1284
Это не логика говорит, а обывательское мышление, человека, которого природа учила миллионы лет бегать от тигра. Два аргумента за то, что там суперпозиция.
1) Уравнение Шредингера работает, еще как работает, и даже эту хуйню объясняет с запутанностью.
2) Неравенства Белла подтверждают, что была суперпозиция.
140 421287
>>1285

>Это не логика говорит


ниправильная логика? Тут простейшая бритва Оккама, не более.
1. уравнение это тоже интерпретация. В конце-концов под суперпозицией вы понимаете два состояния одновременно, разумеется уравнения будут работать при таком условии.
2. вот я об этом выше спрашивал, как именно они подтверждают, по простому ессно. Ну то есть там выпустили 2 фотона, они полетели по световоду в разные стороны, и их детекторы поймали. Как это подтверждает что фотоны летели именно в суперпозиции а не в уже конкретных состояниях, которые определили детекторы?
141 421289
Почему силу магнитного поля меряют через какие-то амперы? Это же абсолютно непрактично, вообще ничего не говорит, много ли 1 тесла или мало. Вот если бы это говорило, с какой силой такой магнит притягивает 1кг железо, сразу было бы понятно кстати, с какой? и какая примерно сила у 100граммового неодимового?.
142 421290
>>1287
В принципе у тебя спин у частиц (у фотонов спиральность) имеют два направления по всем трем осям. Так вот, надо взять детекторы спина, которые могут измерять по всем трем осям, и потом измерять у обоих не просто по z оси, а рандомно по любым. И в случае если у них были уже состояния, то выйдет что несовпадений будет 5/9 от всех измерений. А если не было, то выйдет 50%.
143 421291
>>1290

>измерять у обоих не просто по z оси, а рандомно по любым


хм, ну вот ты измерил что у фотона по 3 осям спин(ы) такой-то. Учитывая что переход от суперпозиции к определенному состоянию даже при измерений одной оси = случайность, то же самое при рождении фотона с уже определенным состоянием, ты не доказал что суперпозиция была в полете между рождением и измерением.
144 421292
>>1291
1) ты не прочитал вторую часть моего сообщения.
2) можно я просто не буду отвечать человеку, который не знает что такое коммутатор? Мне просто лень тратить впустую силы.
145 421293
>>1283

>но не думаю что он может пролежать не портясь - веками.


Ну, не веками, но в сухом темном месте его срок годности во-моему, вообще не ограничен.

С бобовыми-да, хранятся мало и пересаживать каждый год, но это очень как эффективно в плане получаемых калорий и консервировать легко.
146 421303
>>0851 (OP)
Закон Кулона описвает силу взаимодействия между неподвижными зарядами. Напряженность электростатического поля есть сила действующая на неподвижный положительный, еденичный заряд в данной точке. Электрической поле совершает работу, при этом движется заряд, но посему работа равна произведению напряженности на пройденое рассточяние векторно, ведь заряд движется, а напряженнсть определена для неподвижного заряда?
147 421304
Друзья! Как стать такими умными как вы? Это просто капец какой-то.
Решил, например, на курсере изучить курс по теории графов от МФТИ, сначала все понятно:
вот вершины, вот ребро, вот это граф
Графы бывают такие-такие-такие.
А если граф вот такой-то, то это дерево.
Все понятно, хорошо.

а вот теперь РЕШИТЕ БЛЯТЬ ЗАДАЧУ СВЯЗНОСТИ-ХУЯЗНОСТИ ФАКТОРИАЛ ПЕРЕСТАНОВОК ПОМЕЧЕННЫХ ГРАФОВ ПУТЕМ ПЕРЕНУМЕРАЦИИ ВЕРШИН ПОЛНОГО ДВУДОЛЬНОГО ГРАФА К ИТОЕ ЖИТОЕ С ДОЛЯМИ РАЗМЕРА N.

Утрирую, конечно, но как начать понимать как РЕШАТЬ задачи, доказательства всякие, вот это вот всё?

30 лвл, листва.

Спасибо!
148 421306
>>1304
Надо начать с простеньких... мат мышление развивай. Начни олимпиадки решать, для 5-го класса, легко тогда 6-го, и т.д. Это как в играх, просле прохождения кампании не замечаешь босса последнего
149 421308
>>1306
Спасибо! Правда, с математикой у меня вообще никак(даже в таблицу умножения не могу :) )
150 421309
>>1304
Но нахуя? Заниматься фундаментальной математикой могут позволить себе только очень состоятельные люди. За неё не платят. Ну а если платят, то только гикам уровня Перельмана, которых единицы на планете.
151 421311
Большой разрыв или большое сжатие?
152 421312
>>1293

>но в сухом темном месте его срок годности во-моему, вообще не ограничен.



Он окаменеет лет через пять.
154 421321
>>1309
Надоело быть тупым :)
images.jpg8 Кб, 275x183
155 421328
Sup, sci.
Как работает телевизор с 3d? Конкретно тот, что с поляризационными очками. Он сам преобразует 2 картинки в одну чересстрочную, но что конкретно там происходит? Я пытаюсь понять почему этого нельзя добиться программно и вывести такую картинку на монитор.
156 421329
>>1328
с телека идут сразу 2 картинки, но с разной поляризацией. Каждое "очко" пропускает только одну из них для каждого глаза. Программно ты никак не разделишь 2 картинки.
157 421330
>>1328
тебе тоже туда
>>1317
158 421334
>>1329
Но как? Там какой-то особый экран? Частота развёртки? За счёт чего достигается та или иная поляризация картинки на экране?
>>1330
Посмотрел.
159 421338
>>1334
ЖК телевизоры, имеют у себя и так фильтры поляризации, и могут давать определенную поляризацию, если их усовершенствовать. И там просто четные кадры пускают с одной, а нечетные с другой.
160 421340
>>1334
самое простое - это разная поляризация для каждой строки сзади матрицы и отсутствие фильтра спереди(им будут уже сами очки).
изображение.png203 Кб, 855x480
161 421346
Кто-нибудь знает, что за программа на скрине?
Из этого видео https://youtu.be/trJdvW9s0K0?t=435
162 421347
>>1346
А всё, не надо, сам нашёл
163 421354
>>0851 (OP)
ребятУшки. Есть же какие-то книги об истории человечества в глобальном объеме? Видел когда-то книгу коллектива авторов, но забыл название.
Типа вот как у брайсона краткая история почти всего на свете, только другая ЖВ
KaulquappenTadpole2.JPG2,2 Мб, 2634x1464
164 421359
Где-то видел инфу о том будто не все головастики превращаются в лягушек и что типа вырастают какие-то монстры которые жрут других головастиков. Даже и не знаю где спрашивать про такое.
165 421374
Способен ли радий к цепной реакции? То же о радиоактивных продуктах распада урана.
И этот >>1289 вопрос сам на себя не ответит
166 421375
>>1374
Способен ли радий к цепной реакции? То же о радиоактивных продуктах распада урана.
Нет и нет.
167 421377
У алкашей чаще больные дети потому что, грубо: выпил алкоголь, он включает в себя какое-то тупо ядовитое для человека соединение, переварил его в желудке и всосал в кровь этот яд, кровь разнесла его по телу, клетки в половых органах и всём остальном теле всосали яд, некоторые, всосавшие яд, клетки умерли, в некоторых повредилась ДНК. Т. е. и по всему телу виден эффект алкоголя и в половых клетках(яйцеклетки и те что делают сперматозоиды). Больше плохих половых клеток -> выше вероятность больных детей. Теперь вот капитана Америку в фильме сделали: вкололи ему точечно в мышцу какую-то хрень и облучили для стимуляции роста: поскольку это не яд, а спец. состав, в мышцах он вызвал рост, а в половых клетках ничего не изменил, т. е. внешне кеп раскачался, а потомство его будет похоже на него до этой операции.
Правильно понимаю или нет ?
168 421383
Правда ли что мы используем мозг на 10 процентов?
169 421385
Читал у Сапольски "Психологию Стресса". В главе про депрессию в месте, где он объясняет про нейроны, есть предположение, что у аксонов есть рецепторы, которые позволяют определять сколько нейромедиатора было выпущено. В гугле инфы никакой не нашёл. Правда ли что на терминали есть рецепторы или нет?
170 421386
>>1383
Зачем спрашивать гуглящиеся вопросы?
https://ru.wikipedia.org/wiki/Миф_о_10_%_используемой_части_мозга
171 421387
>>1386
просто хотелось услышать мнение кого либо, может здесь есть те кто так не думает
172 421388
Земля плоская?
173 421389
>>1383
>>1386
Все зависит от того, как ты будешь определять понятие используемости мозга. Мне иногда кажется, что многие люди вообще не используют мозг, и им было бы удобнее и безопаснее рождаться с маленькой головой.
Собсна, у челика, который сидит с пивом у телика по вечерами, а на работе выполняет какую-то монотоную работу изо дня в день, в итоге мозг разглаживается.
174 421390
>>1388
Нет, она коричневая и горькая
175 421391
>>1389
спасибо за ваш ответ, достойно)
176 421392
>>1390
пидар ну ты же прекрасно понял о чем я, ну хули ты выебываешься?)))
177 421393
>>1389
Я бы сказал, что именно физическая активность мозга гения и типичного быдлана будет практически одинакова. Только у гения какая-то часть этой активности заточена на абстрактное мышление, а у быдла - на рутинные действия. Или по-другому, процессы в мозгу гения более оптимизированны, и поэтому позволяют производить более сложные вычисления на тех же ресурсах.
Если что, я не нейробиолог, и возможно написал хуйню
178 421394
>>1393
ну а на счет моего вопроса что скажешь>>1383
179 421395
>>1392
Съеби в /b со своим плоскоземельным троллингом
180 421396
>>1395
нахуй иди
181 421397
>>1394
По идее, я на этот вопрос и ответил. То есть, все люди используют мозг полностью, просто кто-то делает это эффективнее. Но в этом смысле понятие "эффективность" слишком размыто, нельзя сказать, что какой-то человек думает в 10 раз "лучше", чем другой. Есть конечно тест IQ, но годится разве что для отличия совсем уж дебилов от нормальных людей
182 421398
>>1397
ясненько, спасибо.
183 421424
Сколько будет весить барабан револьвера длиной в 65 см?
184 421432
>>1375

>Способен ли радий к цепной реакции?


Самый стабильный изотоп радия из всех его изотопов https://ru.wikipedia.org/wiki/Изотопы_радия
имеющий период полураспада 1,6 тыс. лет - это 226Ra.
Существуют ядерные реакции с размножением нейтронов для этого изотопа.
http://www.sciencedirect.com/science/article/pii/0022190260802278
226Ra(n,2n)225Ra and 226Ra(n,3n)224Ra
но образовавшиеся изотопы радия имеют период полураспада
3,6319 days (224Ra) и 14,9 суток (225Ra) - соответственно.
А при захвате нейтрона - радий 224 переходит в радий 225, а этот - назад в 226.

Существует также реакция радия-226 с протонами:
https://www-nds.iaea.org/radionuclides/ra6p5ac0.html
На выходе - актиний-225, который тоже поглощает нейтроны.
Изотоп актиния 226, после захвата нейтрона - распадается медленнее,
изотоп актиния 227 - ещё медленнее.
Наряду с захватом нейтронов 226Ra(n,x)225Ac,
существует также цепочка превращений под действием гамма излучения, при вышибании нейтрона из ядра:
226Ra(γ,n)225Fr→225Ra→225Ac
На выходе - тот же актиний, поглощающий нейтроны.

Все эти реакции цепными не являются, поэтому радий является одной из причин
такого поражающего фактора ядерного взрыва, как Радиоактивное заражение.

Поэтому если в результате попадания бомбы или взрыва радий окажется рассеянным
на значительной площади, то это сделает всю эту площадь биологически вредной.
184 421432
>>1375

>Способен ли радий к цепной реакции?


Самый стабильный изотоп радия из всех его изотопов https://ru.wikipedia.org/wiki/Изотопы_радия
имеющий период полураспада 1,6 тыс. лет - это 226Ra.
Существуют ядерные реакции с размножением нейтронов для этого изотопа.
http://www.sciencedirect.com/science/article/pii/0022190260802278
226Ra(n,2n)225Ra and 226Ra(n,3n)224Ra
но образовавшиеся изотопы радия имеют период полураспада
3,6319 days (224Ra) и 14,9 суток (225Ra) - соответственно.
А при захвате нейтрона - радий 224 переходит в радий 225, а этот - назад в 226.

Существует также реакция радия-226 с протонами:
https://www-nds.iaea.org/radionuclides/ra6p5ac0.html
На выходе - актиний-225, который тоже поглощает нейтроны.
Изотоп актиния 226, после захвата нейтрона - распадается медленнее,
изотоп актиния 227 - ещё медленнее.
Наряду с захватом нейтронов 226Ra(n,x)225Ac,
существует также цепочка превращений под действием гамма излучения, при вышибании нейтрона из ядра:
226Ra(γ,n)225Fr→225Ra→225Ac
На выходе - тот же актиний, поглощающий нейтроны.

Все эти реакции цепными не являются, поэтому радий является одной из причин
такого поражающего фактора ядерного взрыва, как Радиоактивное заражение.

Поэтому если в результате попадания бомбы или взрыва радий окажется рассеянным
на значительной площади, то это сделает всю эту площадь биологически вредной.
185 421451
>>1393
Чтобы нейроны не отмирали и память не терялась когда не используется, происходит какая-то фоновая работа, вроде пингов в сетях. Но фактически при этом нейроны спят, их потребление энергии и питательных веществ минимально.
186 421467
>>1432
Шизик с копипастой - ты опять за свое?
187 421489
>>1467
Вообще-то это несколько паст, а пост был синтезирован в процессе обычного гуглежа.
188 421493
>>1489
Я рад, что по первому пункту (шизофрения) - возражений не последовало. Принятие - необходимый этап излечения.
189 421495
>>1397

>То есть, все люди используют мозг полностью, просто кто-то делает это эффективнее.


>Но в этом смысле понятие "эффективность" слишком размыто, нельзя сказать,


>что какой-то человек думает в 10 раз "лучше", чем другой.


Обычно под эффективностью подразумевается именно энергоэффективность.
Например, производительность систем на базе чипа TrueNorth - оценивается авторами
в 46 млрд «синаптических операций в секунду на ватт».

Разумеется, энергоэффективности можно достичь в процессе оптимизации нейросетевых структур,
формируемых при обучении биологических нейронных сетей,
в том числе и при машинном обучении искусственных нейронных сетей.
190 421496
>>1493
Я тебя помню, просто , оттуда ещё, а теперь - пошёл нахуй.
191 421523
как называется концепция, суть которой в отказе от технического прогресса и в возвращении к природным началам?

и обратный термин я тоже забыл
192 421525
Чтобы меня током ударило из розетки нужно обязательно взяться одним пальцем за фазу, а другим за нейтраль т. е. должен быть хотя бы один контакт и с фазой и с нейтралью ? Или достаточно просто взяться за фазу ?
193 421526
>>1223
Оно рационально и квантуется. Гугли планковское время.
194 421528
>>1526
Ты, видимо, не знаешь, что значит слово квантовать.
195 421529
>>1223
ну атомные часы работают по принципу "количество какогото процесса/среднее количество за секунду". Вот среднее число, вполне себе может быть иррациональным, рациональным и любым. Мы сами же выбираем "что такое секунда".
196 421540
>>1523
Анархо-примитивизм - отказ от научного прогресса.
Обратное - не совсем трансгуманизм, но близко
197 421541
>>1525
Скажем так, при этих ситуациях

>одним пальцем за фазу, а другим за нейтраль


>просто взяться за фазу


- будут сильно разные результаты (т.к. разные эквивалентные схемы), к тому же зависящие от других условий. Но, небезопасно и то и то.

В обычных, бытовых условиях, первое конечно намного опасение, а второе почти безопасно (но не надо это проверять на себе).
198 421543
>>1523

>отказе от технического прогресса и в возвращении к природным началам?


>>1540

>Анархо-примитивизм


- тоже подходит под запрос, но разве не может быть антисциентических/примитивистких концепций не анархистского, а вполне этатисткого толка? (Просто "природной" - тогда постулируется какая то социальная формация, как наиболее "естественная" для человека, после чего придумывается Государство Солнца - котором она должна реализоваться.)
199 421602
Вот есть гравитационные волны. Они тебе спать мешают.
Что будет, если их собрать в узкий "лазер" и пиздануть по пространству?
200 421615
>>1602

>Что будет, если их собрать в узкий "лазер" и пиздануть по пространству?


Буит большая волна.
201 421624
>>0851 (OP)
Анон, поясни пжалки за отличие СТО от ОТО на обывательском языке.
202 421625
>>0893
[ Ага/spoiler ]
203 421626
>>1624
[ /spoiler ]хуй
204 421629
>>1615

>Буит большая волна.


И в чём она будет выражаться? Пространство сплющит вместе со всеми предметами? Скорость света под лучом изменится?
205 421631
Антоны, как можно сделать из маховика подобие сервопривода, чтобы управлять его положением?
Я думаю муфту нацепить и вариатор, чтобы управлять скоростью и крутящим моментом, взлетит ли?
206 421632
>>1631
Управлять положением вала, на который маховик будет передавать вращение*
207 421634
>>1629
Ты проснешься в реальном мире.
208 421640
Что будет со вселенной если исчезнет гравитация?
209 421642
EmDrive - наебалово?
210 421682
Бамп
211 421683
Каким образом теория эволюции объясняет появление крыла у птицы или сложность строения глаза? Если с глазом еще можно сказать, что какие-то формы зачаточного глазного органа, который к примеру реагирует на свет, дает преимущество в выживании. То в случае с "недокрылом" эффект полностью противоположный, т.е. до тех пор, пока крыло не станет эероэффективным, его наличие для животного будет совершенно губительно.
212 421684
>>1683
Ну еще споры ведутся, но мне кажется максимально правдоподобной, что сначала они просто планировали на мелких крыльях.
213 421685
>>1684

>планировали


Это и означает аероэффективность. А тут по сути выходит, что случайно генерируется аеродинамические, пусть даже только для планирования, симметричные крылья, да еще и все это должно балансироваться с хвостом, невозможностью использовать "руки" и устойчивым положением на земле. Мне кажется такую тварь ебало бы все что шевелится.
214 421686
>>1685
Ну не ебёт же.

>>0851 (OP)

А является ли логика объективной? Объективность подразумевает независимость изучаемого от субъекта, объективные истины устанавливаются экспериментом, но нет экспериментов в логике, это чисто рефлексивная субъективная дисциплина.
215 421687
>>1685
Да кстати. А уж с глазом то у тебя какие проблемы, по имеющимся образцам глаз его эволюция восстановлена подробнее все других органов, ну кроме нервной системы может быть которая практически не изменялась. Можно узнать в каких источниках ты искал инфу и не нашёл?
216 421688
В некоторых научно-популярных видео, рассказывающих о двухщелевом эксперименте с электронами с отложенным выбором, предполагается, что электроны будто бы переместились в прошлое, потому что они "сделали выбор" при прохождении через щели. Но разве у электронов есть выбор? Разве они не изначально проявляют волновые свойства, находясь в суперпозиции? При вылете они находятся в суперпозиции, проявляя свойства волны. И так и будут проявлять свойства волны, пока не столкнуться либо с экраном, либо с квантами измерительного прибора и не важно, когда это произойдет. Когда их измеряют после пролета через щели, они не "сделали выбор", они как были в суперпозиции, так и остались, а мы, проведя измерение, разрушаем волновую функцию.
217 421689
Если я собрал простой колебательный контур как а физике в 10 классе по его элементам идёт бегущая волна или нет ? Вообще бегущая волна всегда появляется там где есть переменный ток ? Можно ли сказать что в проводниках и кабелях бегущая волна движется вдоль каждого из них даже тупо в торец провода, а ток бы туда не пошёл т. к. нет замыкания цепи ? Как ведёт себя бегущая волна когда встречает развилку, имеется ввиду из обычных проводов ?
218 421691
https://ru.rationalwiki.org/wiki/Глаз_слишком_сложен,_чтобы_возникнуть_эволюционно

Что-то я немного не понимаю, по словам этих людей, единственное противопоставление теории Дарвина это - воля Аллаха? Что за ебанистическая точка зрения вообще?
219 421693
>>1683
Да заебал. Потихоньку выживали те у кого крылья больше. И так стопицот лет.
220 421695
>>1691
А какой третий вариант возможен? Либо постепенно в результате эволюции, либо сразу по воле Бога(или инопланетян или макаронного монстра - разные сорта креационизма)
221 421697
>>1695
Эволюция по Лаплассу
222 421698
>>1697
Может, по Ламарку всё-таки?
223 421699
>>1683
Вообще-то эволюуия крыла не рассматривается в теории эволюции, а прямо наблюдается в рядах окаменелостей с точной датировкой. Та же пернатость многих динозавров установлена достоверно вплоть до окраса.
224 421700
>>1698
Сорта Ла-
225 421701
>>1624
СТО для ИСО и без гравитации. ОТО для НСО и с гравитацией.
226 421702
>>1695
>>1699
А как дарвин рассматривал само возникновение жизни и сознание? Почему эволюционно вышло так, что сознание это в общем-то некая устойчивая структура, которая в отличии от личности и/или психики не меняется от рождения до самой смерти. Под сознанием в конкретном вопросе я подразумеваю "перспективу первого лица", т.н. "наблюдателя", если использовать филосовсктий термин.
227 421703
>>1702
Считал что жизнь зародилась в горячих водоёмах по типу геотермальных источников, про сознание ничего не писал, писал про психику. цитат я искать не буду, верь на слово.
228 421704
>>1104
Возможно птицы или другие животные рожали своих ципляток живьем либо в какой-то жидкой капсуле, а потом вдруг раз, и жидкая капсула вдруг в результате мутации обросла панцирем и тут БАБАХ, такой способ деления стал невероятно выживабельным, спасал потомство от случайного протыкания оболочки или внешних факторов в первые 2-3 недели после выхода из утробы. И потому резко начал доминировать над предыдущей цепью мутации.
1504371727304.jpg1,7 Мб, 1785x2003
229 421706
Математика объективна или субъективна? Проще говоря является ли математика наукой? Как мне кажется математику от науки в первую очередь отличает не использование индуктивного метода мышления и отсутствие эксперимента, математика не делает общих выводов из частных посылок и затем не проверяет их экспериментально, а ведь без этого наука наукой быть не может.
230 421708
>>1702
Я думаю, что Дарвин вообще не загонялся по такой гуманитарной хуете.
"сознание... некая устойчивая структура... перспектива первого лица..."
Да он бы тебя просто обоссал бы, если бы ты ему эту хуергу на серьезных щщах попытался бы втирать.
231 421709
>>1702
Дарвин НИКАК возникновение жизни не рассматривал.
232 421710
>>1706
Математики и экспериментируют и экстраполируют и проверяют модели тестами. Your argument is invalid
233 421712
>>1710
Какими и как? Первый раз слышу такое. Я один раз протестировал догонит ли ахилес(математики сказали что вместо ахилеса может быть кто угодно) черепаху или нет, в итоге догнал и перегнал, еще и апельсин смог порезать только на конечное кол-во частей, сказал математикам что их гипотезы е подтверждаются опытом, так они меня на смех подняли сказав что бы я не лез со своей технарской эмпирикой в их царство вечной дедукции и мысли.
234 421713
>>1712
Ахиллеса и черепаху придумал философ полемик. Там нарочно доказательство так построено, чтобы выглядело будто он не догонит никогда, к математики же оно мало отношения имеет. Помню выводил формулы, чтобы понять в чём тут дело.
Что можно разрезать апельсин никто не говорил говорили, говорили про его абстрактный эквивалент.
235 421714

>>42171


Это просто троллоло голимое.
that's bait.png
236 421715
>>1689
бамп вопросу
237 421717
>>1713
Хуясе у нас тут математик мирового уровня, чёж ты свои формулы то в какой нибудь журнальчик не послал? А то математики так и сидят.

>Что можно разрезать апельсин никто не говорил говорил


>Математики и экспериментируют и экстраполируют и проверяют модели тестами.


Мопед не мой, я только объявление разместил?
303.jpg60 Кб, 600x907
238 421718
Сап, Sci. Сегодня у себя в биофаке спорил с одногруппниками. Речь шла о простых и сложных углеводах. По их словам, простые углеводы-это моносахариды, а сложные-это всякие олиго и полисахариды. Они сказали,что бургер из мака- фастфуд из быстрых углеводов. Но почему? Углеводы в бургере- крахмал, это сложный углевод, который является смесью полисахаридов. Получается, бургер жратва из сложных углеводов. В ответ они что-то невнятное промычали про то, что в результате обработки пищи, крахмал расщепляется, но пруфов не предоставили. Хотя понятно, что это пиздеж, так как крахмал расщепляется под действием фермента слюны во рту, достаточно хлеб рассасывать 10 минут и он будет сладкий. Прошу объясните этот момент, аноны
239 421719
>>1688
Почитай на английской вики про "декогеренцию". Там расписано неплохо.
240 421720
>>1706
Математика просто развивает что-то из аксиом. Фактически "результат", это различные предположения в виде аксиом. Мы просто сразу себе оставляем готовые решения.
241 421721
>>1718
"Скорость" углеводов зависит от того, насколько быстро они усваиваются, повышая уровень глюкозы в крови, так что твои одногруппники отчасти правы.

>крахмал расщепляется, но пруфов не предоставили


Расщепляется. Для чего, по-твоему добавляют дрожжи?
Отчасти надо заметить, что у крахмала тоже высокий гликемический индекс.
242 421723
Какой вообще смысл заниматься в области не связанной в ИИ? Ведь очевидно, после того как ИИ будет запущен он в один миг(давайте забудем про время затраченное такую формальность как эксперимент) "узнает" всё что великие учёные мужи высирали из себя в течении последних 400 лет. В следующий миг он узнает всё что узнали бы другие учёные мужи за последующие тысячу лет и так далее. Это пустая трата времени по сути.
243 421724
>>1723
1. Возможности ИИ ограничены железом, на котором он запущен, т.е. он должен будет ещё хуй знает сколько проработать, прежде чем подвинет науку вперёд.
2. Необходимые эксперименты он тоже сам поставит за доли секунды, да?
3. Даже если все учёные мира массово пойдут учить нейронки, полноценный ИИ вряд ли будет создан в ближайшее время. Зато вся остальная наука не будет развиваться.
4. Всегда существует вероятность получить в итоге как минимум 42 хуй, как максимум нечто недружелюбное/некооперирующееся.
Вывод: ИИ - это, конечно, интересная и перспективная область, но явно не приоритетная.
1502220542658.png26 Кб, 500x500
244 421725
>>1724
В чём прикол отвечать на пост который ты даже не читал? Это какой то запредельный уровень школовства что бы не прочитать пост в 2 строчки и ответить на него постом который длиннее.
245 421726
>>1709

А это что? фальсификация?

https://www.darwinproject.ac.uk/letter/DCP-LETT-7471.xml
246 421727
>>1725
Что заставляет тебя считать, что я не читал твой пост? Конкретно.
1502728243369.png48 Кб, 132x140
247 421728
>>1727
Хотя бы это.
>>1724

>Необходимые эксперименты он тоже сам поставит за доли секунды, да?


>>1723
(давайте забудем про время затраченное такую формальность как эксперимент)
248 421729
>>1728
Окей, я долблюсь в глаза. Но это не отменяет остальных трёх пунктов из моего поста выше.
249 421730
>>1723

> В следующий миг он узнает всё что узнали бы другие учёные мужи за последующие тысячу лет и так далее


С чего бы это?
250 421733
>>1719
Декогеренция - это коллапс волновой функции или нет? Если нет, в чем разница?
251 421734
Анон. Такая хуйня. Один нарк мне вчера затирал, что скорость света нихуя не постоянная. Что, мол, фотон стартует не сразу со скоростью с, ему надо разогнаться, пускай даже за миллиардную секунды, что есть V0=0 и V1=c, и есть разгон между ними. Я не шарю в столь тонком вопросе, но помоги мне его переубедить, если это возможно!
252 421735
>>1734

>но помоги мне его переубедить


Просто нассы на него ночью, пока он спит.
sage 253 421736
>>1734
>>1734
безмассовое тело, никогда не имеет скорость отличную от скорости тела, увы, но это так.
254 421737
>>1689
Ещё бамп.
255 421738
Что за новшество и обосрамсы, я читаю с утра. Поясняйте.
http://actualnews.org/exclusive/201104-uchenye-paradoks-v-teorii-eynshteyna-stavit-pod-ugrozu-fizicheskuyu-kartinu-mira.html
256 421739
>>1733
Про коллапс говорили почти 100 лет назад, сейчас разобрались в чем дело. Она не коллапсирует, просто влияет на окружение( и на тебя)
257 421740
Напряжение 1,5 В безопасно для человека. А если представить себе теоретический повышающий трансформатор с 1,5 В на 10 кВ? То эти 10 кВ тоже будут безопасны для человека?
258 421741
>>1740
Переменный или постоянный?
259 421742
Переменный.
slide19[1].jpg61 Кб, 960x720
260 421753
>>1740
Если подключить повышающий трансформатор к источнику высокоамперного постоянного тока
не важно пусть даже если там будет 1,5В - может ёбнуть хорошо.
https://www.youtube.com/watch?v=YMmfYHG1gfA
КПД трансформатора зависит не только от напряжения, но и от тока.
Я как-то подключал повышающий трансформатор к батарейке обычной,
и чисто ради интереса держал двумя пальцами контакты с повышающей обмотки.
В момент контакта - хорошо так трепало, как пьезоэлементом,
но убить не могло - потому что ток там маленький был.
Короче что-то наподобие неприятной миостимуляции.
261 421771
Почему ученые считают, что гравитон реален, если гравитация - искривление САМОГО пространства-времени? Чтобы можно было проквантовать гравитацию? А других подходов нет?
ylQarZ5wjU.jpg98 Кб, 807x538
262 421863
>>1771

>А других подходов нет?


Есть.
263 421864
>>1734
Точно не известно, разгоняется он или сразу вылетает с такой скоростью. Скорее нечто среднее. Разгоняться фотон умеет, например, в стекле он движется чуть медленнее чем в воздухе, а значит при выходе фотона из стекла в воздух идет разгон.
264 421865
>>1736
Пруфы на эти два тезиса?
265 421871
>>1865
СТО и ОТО, такие подойдут?
266 421878
>>1871
Нет конечно. Манятеории, основанные на крайне сомнительных гипотезах не могут являться пруфами.
267 421879
*
268 421881
>>1878
Ну тогда иди нахуй, так как пруфов у этих теорий предостаточно.
269 421886
>>1864
Ты не прав анон. У фотона скорость в веществе меньше чем в вакууме не потому, что он замедлен, а потому, что он постоянно поглощается и излучается атомами вещества по пути движения фотона и собственно сумма этих моментов между поглощением и излучением и дает среднею меньшую скорость.
270 421888
>>1886
Как он может поглощаться и переизлучаться если у него меняется направление при входе в вещество. Идет именно замедление, что приводит к повороту частицы, когда одна ее сторона движется медленнее чем другая. К тому же атомы переизлучают в рандомном направлении и с определенной энергией.
271 421890
>>1888

>у него меняется направление при входе в вещество


Тут ты прав. Но дальше нет. Прочитай про физический механизм дисперсии света. Например http://www.femto.com.ua/articles/part_1/1051.html
(Ссылка из вики)
Там электроны рассматриваются как затухающие гармонические осцилляторы, направление излучения которых складывается с полем исходной волны.
272 421891
Какой у фотона объём?
Как выглядит мир с точки зрения фотона?
Что определяет, в каком направлении и с какой поляризацией частица испустит фотон?
На научпоп-передачах при наглядном объяснении искривления пространства часто используют образ тонкой мембраны/сетки. Однако, такое двумерное "пространство" выгибается в третье измерение, и так искажается. Куда выгибается искривлённое пространство вселенной?
Чем дальше галактика, тем больше её красное смещение. Чем больше красное смещение - тем больше скорость. Но чем дальше галактика - тем младше мы её видим. Но чем ближе галактика - тем меньше (в среднем) красное смещение. Значит, меньше скорость. Но чем ближе галактика - тем свежее излучённый ею свет. Получается, что ускорение, с которым разлетаются галактики - падает?
273 421894
>>1891
1. Никакой. У фотона нет объема.
2. Никак. У фотона нет глазок.
3. При спонтанном испускании фотона, направление в общем случайно, при вынужденном - совпадает с направлением индуцирующего.
4. Само в себя, лол. Не так уж просто представить, ага. Как вариант, можешь представить "выгибание" как плавное "увеличение плотности пространства", или как "выгибание" ещё в одно, виртуальное измерение.
5. Не понял твоей логики. При чем здесь свежесть света? Да, и вывод у тебя неверный. В данный момент считается, что скорость разбегания галактик, а-ля скорость расширения Вселенной растет.
274 421898
>>1863
Лол.
А если все же окажется, что гравитонов не существует, как это повлияет на физику?
275 421899
Если над пустынями на ГСО развернуть ик-отражатель (от солнца), сильно ли это охладит? Приедут ли дожди с океана? И насколько сам отражатель будет греться?
276 421900
>>1890

>физический механизм дисперсии света


Там не физический механизм, а немножко ущербное модельное представление.
277 421901
>>1894

>У фотона нет объема.


Есть. Это же не точка. Фотон занимает вполне определенное место в пространстве, иначе бы его не было.
278 421902
>>1894

>направление в общем случайно


Немного поправлю. Направление псевдослучайно, то есть определяется всем комплексом условий. Случайностей во вселенной нет.
279 421903
>>1891

>Куда выгибается искривлённое пространство вселенной?


Лучше и физически вернее это понимать не как искажение пространства, а как искажение среды в пространстве. Релятивисты выдумали слишком неудобную модель, аж жалко их, как они насилуют свой мозг.
280 421907
>>0851 (OP)
Анон, возможно существование двойной гравитации? Одновремено влияющих на тело? Естественно искусственно.

Предположим есть некий полый цилиндрический объект. Цилиндр находиться параллельно поверхности земли. Внутри него тело. На тело, с помощью искусственной невесомости, не влияют силы притяжения земли, но влияют внутренняя гравитация, т.е. тело имеет точку притяжения в основание цилиндра. Грубо говоря, внутри цилиндра гравитация перпендикулярна гравитации поверхности земли.

Сразу напрашивается решение второго условия магнитами, но если возможность обойтись без них?
281 421908
>>1902
Ататата... аккуратнее..А то ты сейчас выдумаешь скрытые параметры, которых нет. Квантами если считать, то он излучается во все направления сразу. А вот уже то, что мы обнаруживаем его в каком-то конкретном месте, это банальный эффект наблюдателя срабатывает. Если ты смотришь на далекую звезду, то от неё до тебя "долетает" довольно много "фотонов", которые при измерении оказываются тут, но еще больше, которых при измерении вообще не оказывается.
mass.webm684 Кб, webm,
600x600, 0:04
282 421912
>>1891

> Куда выгибается искривлённое пространство вселенной

283 421915
>>0851 (OP)
Почему до сих пор не разработано ни одного фармацевтического препарата для специфической терапии вирусных заболеваний, которые относят к ОРВИ? Будь это риновирусная, аденовирусная инфекция, что-то ещё - не принципиально - они не лечатся. Понятно, что родов таких вирусов десятки, видов многие сотни, а серотипов - тысячи (поэтому против ОРВИ не вакцинируют, как я понял). Понятно, что в современных условиях все эти инфекции легко проходят сами и имеют шанс осложнений на уровне статистической погрешности. Но абсолютно все болеют ОРВИ по несколько раз за год (что само по себе неприятно, это снижение эффективности труда) и, получается, вирусы имеют огромную циркуляцию в популяции и, получается, широчайшую базу для лампового мутационного процесса. Я не вучоный, но А ВДРУГ какой-нибудь, казалось бы, безобидный кашлевирус приобретёт мутацию, которая позволит ему поражать клетки вне слизистых оболочек носа например? Все опасные вирусные заболевания были побеждены только потому, что их возбудители не отличались разнообразием, и вакцинация резко сократила циркуляцию вируса. Если быковать начнёт что-то вроде кашлевируса, мы же все вымрем без возможности разработки вакцины и без протививирусных препаратов.
284 421916
>>1894

>1. Никакой. У фотона нет объема.


Но у него есть длина волны. Скажи, а какой объём имеют другие элементарные частицы?

>2. Никак. У фотона нет глазок.


А в ИСО, связанной с фотоном, как выглядит мир?

>4. Само в себя, лол. Не так уж просто представить, ага. Как вариант, можешь представить "выгибание" как плавное "увеличение плотности пространства", или как "выгибание" ещё в одно, виртуальное измерение.


Тогда вот какой вопрос. Если Лоренцово сокращение длин - тоже эффект пространственного искажения, то что происходит в этом случае?
И ещё - можно ли прыгать по искривлённому пространству, как на батуте?

>Не понял твоей логики. При чем здесь свежесть света?


При том, что удалённые объекты мы видим такими, какими они были некоторое время назад. Чем дальше от нас объект - тем больше ускорение, однако чем он дальше - тем менее свежие данные мы о нём получаем.
285 421918
>>1916
В исо связанной с фотоном мы не можем перейти, там сингулярность
286 421919
>>1918
То есть для фотона весь мир - это сингулярность? И взаимодействие фотона с элементарной частицей невозможно описать с точки зрения фотона?
287 421920
>>1919
Скорее всего нет. Но я не знаю квантовой теории поля, может там что есть.
288 421921
>>1920

>Скорее всего нет.


Что делать-то тогда?
289 421922
>>1921
Не описывать что-то в системе фотона?
290 421923
>>1922
Но как тогда согласовать происходящее в разных системах отсчёта?
291 421924
>>1923
Нахуя это делать в системе фотона то?
292 421925
>>1924
С системой фотона. А то у нас получаются системы, привязанные к объектам реального мира, но в которых ничего невозможно описать.
293 421926
>>1925
А что ты описать то хочешь?
294 421927
>>1926
Хоть что-нибудь. Тут ведь пишут - "сингулярность".
295 421928
>>1927
Но для этого не надо же прям переходить в систему с фотоном. Я просто хз откуда такое желание прям.
296 421929
>>1928
Хочется узнать границы применимости теории.
297 421930
>>1929
Ну границы как раз и есть, скорость света. И всякие места с кривизной пространства сингулярной, типа черных дыр.
298 421933
>>1916

>А в ИСО, связанной с фотоном, как выглядит мир?


Выглядит как в мир в котором не идет время, всё застыло, т.к. часы у фотона остановлены. И для него перемещение в пространстве происходит мгновенно.
299 421934
>>1933
Проблема взгляда фотона в том, что все объекты в его мире, имеют скорость света, а значит и бесконечную массу и энергию. Но с первого взгляда именно так, да и пространства нет для него, у него всё сплюснуло.
300 421937
>>1930
У относительности применимость только околонулевые скорости, то есть смысла в ней нет. Всякие высокоскоростные эффекты она не способна верно предсказывать.
301 421938
>>1937
Ну ебтвоюмать, не обсирался бы так? Хорошо предсказывает слияние черных дыр. Отлично дает все траектории на БАКЕ. ты блять что вообще имеешь ввиду?
302 421940
>>1933
>>1934
значит ли это что через фотон можно передавать информацию в прошлое/будущее? Раз для фотона не течет время, можно, к примеру, сменить его поляризацию в конце пути, а перед этим считать ее в начале.
303 421941
>>1940
Кек... нет. Так как наше "для фотона не течет время" довольно на пальцах. Ибо там уравнения в пизду полетели, и ничего нельзя сказать конкретного. Плюс тебе кванты не дадут так выебываться.
304 421942
>>1941
ну вот опять, там уравнения полетели, там кванты не дают. Наука, хуле.
А вообще, все это сильно смахивает на частоту обновления в симуляции. Представь если бы действие происходило не ирл а, скажем, в Майнкрафте(за пример взято то что всем известно). функционально связанные блоки красной пыли или механизмов будут последовательно обрабатываться за 1 игровой такт. Разумеется для такой "волны"(череды обновлений блоков в 1 такте) весь мир застынет по-настоящему, ибо обработчик мира занят.
Исходя из такой аналогии можно строить охуительные теории как про Матрицу и эфир с "частотой обновления", так и просто учесть тот факт что все есть э/м волна(фотон, протон, электрон, etc), а значит все подчиняется законам распространения э/м излучения, в т.ч. и критической скорости. Но это не значит что что-то не являющееся э/м волной не может двигаться быстрее. Что опять-таки приводит к теории эфира, то есть некой среды которая и ограничивает распространение волн.
305 421943
>>1942
Кекеке... наука и не претендует же на истину. Так как найти закон который "описывает всё", в принципе не суждено. Но разные теории с отличной точностью, и со своими разумными границами применимости круто подходят. Про майнкрафт это немного плодит сущности. Так как удивляться, что в реальности есть что-то похожее как в симуляции, очень странно. Ибо
1) Законы видимо везде работают одинаково, по крайней мере стараются из этого принципа выводить всё
2) Ну черт игру создавали то люди, а они живут в реальном мире, и ощущают себя вполне конкретно.
3) Компьютер тоже сам в реальном мире, и он работает на этих законах), было бы странно если в игре реализовалось что-то такое, что было бы сложнее чем в реальном мире. Как тогда это надо было бы реализовывать?)
Короче, просто игра похожа на реальность, а не наоборот.
А по поводу "что-то тормозит". Так это же поле Хиггса. Оно частицам с массой дает массу, это это поле не дает некоторым частицам спокойно разгоняться. Я конечно не знаю квантовой теории поля, но пишут так.
Это еще философы описывали, что мы будем в вечном круге познания двигаться от точечных к общим, и опять переосозновать так до бесконечности. Вон изначально думали свет корпускулярный или волновой, с каждой стороны были пруфы, и все спорили. реальность где-то посередине оказалась. Так же с эфиром, сначала пытались вводить, поняли что нифига не выходит, и оставили физический вакуум пустым. Потом проквантовали поле, и выяснилось, что он не пуст, там особая среда. Опять же что-то среднее между эфиром и пустотой. Мы будто по спирали движемся к предельной точке реальных законов.
306 421944
>>1938

>слияние черных дыр


Слияние гипотетической хрени, которое не проверить.

>все траектории на БАКЕ


В бак вложено слишком много чтоб идти против цели вкладчика.
307 421945
>>1944
шапочку из фольги надень. У тебя и Максвелла законы не работают? И линзирования нет? И красного смещения? И часы на орбите не показывают замедление времени предсказанное ОТО?
308 421946
>>1945

>Максвелла законы


В них много чего не заложено и они содержат ошибки, поскольку противоречат классике.

>линзирования


Объясняется обычным преломлением на среде распространения.

>красного смещения


Результат потери энергии фотоном на распространение сквозь толщу среды.

>часы на орбите


Там часы можно подобрать, поскольку ход часов зависит не только от времени, но и от, внезапно, принципа действия часов. Например, маятниковые часы в невесомости вообще стоят, вывод - шок, время там остановилось.
309 421947
>>1946
Хорошо... давай тоже самое, только теперь на формулах. Если что, и часы и всё о чём я писал это электродинамика. ты видимо хочешь разные законы для одних и тех же вещей, зеленый.
зачем кстати плодить их, если точность предсказаний ОТО отличная?
310 421948
Ну, большой взрыв, например. Ещё понятна логика расширения чего-то большого из чего-то маленького. Ну, откуда они узнали вообще про детали, нуансы? Про все это планковское время, про то, что на такой-то миллиардной доли секунде появился водород, на такой-то гелий? И Про температуру? Максимум можно сказать: "Было горячо. Наверное." Этак они скоро допиздятся, что скажут вообще, что было до большого взрыва.
311 421949
>>1948
Они же на коллайдере создают такие энергии частиц, которые соответсвуют огромейшим температурам. Изучают всё это с помощью компов статистики матана. Ты заметил что как-то после ядерной энергетики куда-то физика частиц пропала будто бы, даже в научпопе почти нету. Хотя если на то пошло, там дофига наоткрывали и наделали, просто всё настолько уже сложно стало, что просто никто туда не лезет. Они проверили экспериментально все формулы проверили их границы применимости, посчитали их точность. Из них же смоделировали время в обратную сторону, просто поменяв знак в законах. И докрутили до момента, когда температура начальной вселенной была как температура которую они получили на БАКе, типа дальше теория еще может считать, но они не проверяли, работает ли она адекватно при больших температурах.
312 421950
При варке как на "малом огне", так и на "большом огне" кусок мяса окружает вода с одной и той же температурой 100°С. Тогда почему "на большом огне" варка идёт быстрее (ну по крайней мере так считается)?
313 421951
>>1950
Ты стал правильно понимать фишку, так просто считается.
314 421952
>>1950
если температура 100°С то разницы нет, но обычно она меньше, и "кол-во огня" позволяет достигнуть кипения быстрее, ну и поддерживать эту температуру.
315 421953
>>1947

>давай тоже самое, только теперь на формулах


Может тебе готовую численную модель теории всего подогнать?

>точность предсказаний ОТО отличная


Только во влажных фантазиях исповедующего релятивизм.
316 421954
>>1943

>Про майнкрафт это немного плодит сущности.


я ж не говорю что это реальная теория, просто аналогия. Плодить сущности можно и нужно если остаются непонятные моменты в основной гипотезе.

> просто игра похожа на реальность, а не наоборот.


но не в этом случае. Я взял свойство, присущее исключительно симуляциям, при чем синхронным и однопоточным. В реальности же нет центрального обработчика(процессора), каждый объект, будь то планета или частица, живет своей жизнью асинхронно с другими. Считается что нет общей среды и что все объекты летают в "небытие", которое не может тормозить или иметь вязкость. Однако тот факт что все частицы не могут распространяться быстрее определенной границы уже ставит под сомнение данную теорию. Тут либо сам свет не может быстрее двигаться(электрическое и магнитное поля имеют ограниченную скорость распространения), либо их что-то тормозит.Учитывая выводы из ОТО об искривлении пространства, дело похоже таки в эфире.
В квантовой же физике вообще происходит некая магия. Частица может тупо быть во всех местах одновременно, но даже это свойство(которое вроде как абстрагирует частицу от эфира) не может дать частице увеличить свою скорость путем "телепортации" вперед себя.
317 421955
>>1953

>Может тебе готовую численную модель теории всего подогнать?


Ты серьезно? Ты думаешь её можно сделать?
Призываются физики! 318 421956
Представим, что у меня есть в распоряжении бесконечных размеров комната с одной лампочкой, которая видна на любом удалении от нее и зеркалом напротив лампочки. Ширина комнаты не особо важна но главное длина. Она может составлять до ∞ км. Но не суть. Если загорится лампочка на одном конце комнаты, то ее отражение в зеркале появится через время, ограниченное скоростью света?
319 421957
>>1956
кек... ну конечно
320 421958
>>1956
такой опыт ты можешь лично наблюдать взглянув на ночное небо.
321 421959
>>1958
Ты про то что свет небесных тел много дней а то и месяцев до нас доходит? Ну а вообще я понял в чем суть моего вопроса
322 421960
>>1933

>часы у фотона остановлены


Тогда с точки зрения фотона в мире ничего не происходит? Скажи, а процесс взаимодействия этого самого фотона с электроном тоже неописуем?
И вот ещё какой момент. Если скорость света - величина постоянная во всех исо, то и в фотонной исо скорость любого тела относительно фотона будет одинаковой и постоянной. Однако в нашей системе различные тела имеют различную скорость. Как это согласовать?
323 421961
>>1915
Почти все вирусы присутствуют в легких/полости рта. Если начать их травить, то будет как раз

>кашлевирус приобретёт мутацию, которая позволит ему поражать клетки вне слизистых оболочек носа


потому что прием селективного препарата вносит фактор существенный фактор давления естественного отбора. И тогда

>широчайшая база для лампового мутационного процесса


становится не эволюционным шумом, а причиной экспансии новых штаммов за счет эффекта бутылочного горлышка.
324 421962
>>1960
Для фотона понятие времени и расстояний бессмысленны, из-за этих эффектов:
https://ru.wikipedia.org/wiki/Релятивистское_замедление_времени
https://ru.wikipedia.org/wiki/Лоренцево_сокращение
Так что в ИСО, связанной с фотоном, вообще ничего не существует
325 421964
>>1954
Читни про интеграл по траэкториям, там можно быстрее скорости света.
326 421966
Поскольку я не нашел доски с философией или чем то подобным, а философия это вроде как тоже наука задам тупой вопрос здесь. Почему некоторые человеки не могут просто так взять и включить пресловутый похуизм? Можно ли у кого либо этот самый похуизм включить и если да, то как?
327 421995
>>1955
Да. Это цели физики. 1. Узнать как на самом деле ведет себя материя во вселенной. И 2. Придумать несколько численных моделей разной степени точности для упрощенного расчета поведения материи.
328 422021
329 422022
Двощ, помоги. Хочу из цинковой пасты намутить сульфат цинка, но столкнулся с двумя непонятками.
1. Прореагирует ли вся кислота и весь ZnO? Думаю, выход - кипятить@перемешивать, но хз, вазелин очень вязкий.
2. Как расчитать нужное количество серной кислоты. У меня есть доступ только к 10% электролиту из автомага, пойдет ли такой?
нищеброд, забывший школьный курс химии
330 422024
>>1995
Мы не можем её найти. Теория всего недостижима, из-за самой математики. Теорема Гёделя
331 422033
>>1959

> свет небесных тел много дней а то и месяцев до нас доходит


А ты оптимист.
332 422037
>>0851 (OP)
Лучшие страны/места для астронома. Чтобы наблюдать небо. Живут на ебучих уральских горах, вечная серость и срань, небо нихуя не видно 10 месяцев из 12.
Хочу на эти ваши звезды и планетки глядеть.

Максимум, что я видел в своей жизни-парочка дохлых звезд иногда+луну.
333 422038
>>2022
Ой бля, не нашёл в ютубе реакцию взаимодействия серной кислоты с оксидом цинка,
но нашёл обратную реакцию получения оксида цинка из сульфата - через карбонат:
https://www.youtube.com/watch?v=rWnVOZEc6e0
Но тебе же надо сульфат...
Нашёл вот что:
1. ZnO+2HCl -> ZnCl2+H2O;
2. ZnCl2+H2SO4=ZnS+HCl;
3. Во влажном воздухе сульфид цинка окисляется до сульфата; при нагревании на воздухе образуется ZnO и SO2.
А ещё нашёл вот что вот что: https://www.youtube.com/watch?v=DyMI1AnaVBA
ну тут надо металлический цинк, его можно получить из оксида - так:
ZnO + C = Zn + CO
2ZnO + C = Zn + CO2
но это надо прокаливать с углём, и там тогда будут пары цинка, а их придётся конденсировать.
334 422039
>>2038

>Во влажном воздухе сульфид цинка окисляется до сульфата


Реакцию окисления кислородом не нашёл, их много в гугле, понять не могу какая норм,
поэтому на ещё вот это тебе - перекись вроде лучше окисляет:
4H2O2 + ZnS → ZnSO4 + H2O
http://chemequations.com/ru/?s=H2O2+++ZnS+=+ZnSO4+++H2O&k=1&ref=vr
335 422043
>>2038
Спасибо, анон. Твой длиннопост дал понять, что я плохо искал. Погуглив полчаса я пришел к этому.

Pharmaceutical grade zinc sulfate is produced by treating high purity zinc oxide with sulfuric acid:
ZnO + H2SO4 + 6 H2O → ZnSO4(H2O)7

Далее подсчитал молярную массу (все же не все из химии забыл:
81 + 98 + 6x18 = 121 + 18x7
И как я понял, на каждый грамм оксида мне нужно 1,2098 кислоты, и выйдет 1,98г. сульфата.
Пойду крафтить, надеюсь ничего не упустил.
336 422044
>>2043
Я просто не видел в ютубе реакции оксида цинка с серной кислотой,
оттого и пошёл окольным путём через эти хлориды и сульфиды.
И подумалось мне, что аверное лучше чистый сульфат получить прямым взаимодействием цинка с серной кислотой.
Для получения цинка из оксида, можно было бы растворить оксид в соляной кислоте - эта реакция точно пройдёт,
а потом восстановить цинк на угольных электродах электролизом, и скормив раствору серной кислоты.
Но если у тебя получится получить сульфат цинка прямым растворением - напиши здесь, ок?
337 422050
Аноны. У меня появилась идея запустить ракету. Я уже определился с материал из какого она будет сделана, но еще не определился с топливом для неё. Какое топливо лучше использовать? На ум приходит только керосин, но хотел бы услышать ещё варианты. Еще прошу помощи с самим пояснением заправки ракеты топливом. И каким образом я могу запустить её.
338 422053
>>2024
Математика это не физика, это всего лишь инструмент для численного моделирования.
339 422056
>>2053
а другого инструмента нет, так что сосём.
340 422057
>>2050

>Какое топливо лучше использовать?


https://ru.wikipedia.org/wiki/Ракетное_топливо#.D0.A5.D0.B8.D0.BC.D0.B8.D1.87.D0.B5.D1.81.D0.BA.D0.B8.D0.B5_.D1.80.D0.B0.D0.BA.D0.B5.D1.82.D0.BD.D1.8B.D0.B5_.D1.82.D0.BE.D0.BF.D0.BB.D0.B8.D0.B2.D0.B0
Там не только топливо надо, но ещё и окислитель, чтобы ровно и стабильно летела.
Всё зависит от тоннажа. Можешь запустить феерверк - чем не ракета тебе?
Или иди на какие-нибудь военные учения, там можешь спросить сколько стоит у них тестовый запуск ракеты
с какой-то там установки залпового огня по различным учебным целям.
А если вообще большая ракета и на орбиту хочешь вывести - иди на космодром.
Вон SpaceX не только пускают ракеты, но и сажают их назад на морские платформы. Пару раз получилось.
341 422058
>>2022
добавь просто до полного растворения частями, вот и всё. потом прокалишь, серка улетит.
 .jpg8 Кб, 203x247
342 422059
Шах и мат, защитники круглой Земли.
343 422061
>>2059
Это Луна вообще-то, это они её так погрызли.
147988733218682539[1].jpg58 Кб, 600x600
344 422062
345 422064
Какова вероятность того, что Земля, её спутник и Солнце настолько отдалены друг от друга, что лунный диск идентичен солнечному и при этом Луна всегда повёрнута одной и той же стороной к Земле, вращаясь вокруг неё?

Почему звёздное небо выглядит всегда одинаково, если с одной половины планеты мы можем видеть только одну его часть, а с противоположного другую, но вращение и оборот вокруг Солнца никак этого не меняют?
Вопросики 346 422065
1. Какой отклик у зеркала?
2. Почему люди не занимаются селекцией самих себя? Что может быть лучше, чем создавать людей изначально без всяких уродств и дефектов? Реквестирую данные о результатах такой селекции в Третьем Рейхе, на Википедии вода какая-то.
3. В чём выгода предприятия, если оно выпускает акции? Ну то есть с приобретателями понятно - они могут выиграть на росте таких активов и получать дивиденды, а у компании только единовременный приход, так?
347 422070
>>2057

>Или иди на какие-нибудь военные учения, там можешь спросить сколько стоит у них тестовый запуск ракеты


А самому мне нельзя её запустить?

>Там не только топливо надо, но ещё и окислитель, чтобы ровно и стабильно летела.


Это понятно, но мне было важно узнать каким образом это все заправлять. Как я понимаю, нужно сделать отдельные отсеки для топлива и окислителей? И каким образом это всё контролировать?
15.jpg33 Кб, 1024x576
348 422071
>>0851 (OP)
Возможно ли технически вознестись? Хоть в каком-либо виде. Не спрашивая за сегодняшнюю науку
349 422076
>>2070

>каким образом это всё контролировать?


Конструкционным и сбалансированным образом. Ну там форсунки всякие чтоб смесь была равномерной и такое прочее...
Если ракета ёбнется - лучше её взорвать в небе, дать упасть на открытой местности или же навести на какую-нибудь ложную учебную цель.
Можешь делать это на каком-нибудь полигоне или на острове, если вообще термоядерный космос.
Ну и камеру не забудь взять, чтобы пруфы завезти.
350 422077
>>2071
Как-то вероятностно-предопределённо подключиться к Боженьке, очевидно же.
351 422078
>>2077
Только надо ещё и правильно.
352 422079
>>2077
не понял
353 422080
>>2079
Возможно потом поймёшь, если будет на то Его воля.
354 422081
>>2070
Ты сначала поясни, какую конкретно ракету ты хочешь: высота, полезная нагрузка, тип двигателя, после этого можно что-либо советовать, может тебе хватит твердотельного топлива.
355 422082
>>2065
2. Где-то слышал такое объяснение:

Пусть носители aa - обычные особи, оставляющие после себя в среднем 2,5 ребенка, аА - тоже здоровые, но усиленной ебливости(в случае самок - плодовитости), поэтому у них по 5 детей в среднем, а носители АА не доживают до половой зрелости ( ну так вот природа решила). Тогда якобы "плохой" ген А никогда не уйдет из генофонда, потому что его распространяют аА-шники.

Ещё погугли про ниггеров, которые были генетическими уродцами и болели серповидной анемией. После очередной эпидемии малярии их выживало больше, за счёт чего их гены тоже не выкашивались.
14940008828190.png60 Кб, 1070x884
356 422092
357 422096
>>2081
Откопал тред в архиваче. Один анон тоже хотел запускать ракеты, но я оттуда и взял технические характеристики.
Материал-алюминий
Внутренний диаметр 15 мм, длина 26 мм.

> тип двигателя


У меня с этим и возникает вопрос. Хочу чтобы вышло дешево, но нужно чтобы взлетело.

>высота


Если ты про высоту на которую я хочу её вывести, то мне не принципиально. Главное, чтобы она могла взлететь.
>>2076

> Ну там форсунки всякие чтоб смесь была равномерной и такое прочее...


С этого места подробнее. Ничего не понял.

>Конструкционным и сбалансированным образом.


Тут тоже хотелось бы пояснения.
Безымянный.png8 Кб, 378x811
358 422100
Котаны, помогите школотрону решить простую задачу.
Электрический бойлер для упрощения задачи предположим, что он выключен объемом V=100л наполнен водой температурой T1=80°C, и подключен к системе городского водоснабжения с температурой T2=15°C. Какой объем воды для наполнения ванны мы можем получить, если желаемая температура равна 35°C?
359 422102
>>2065
Сейчас занимаются лучше. Генной инженерией самих себя. Погугли недавнюю статью из Нэйчар, там ребята вырезали и поменяли ген, отвечающий за заболевание сердца.
360 422103
>>2100
Если смотреть в приближении что теплоемкость воды в пределе 15-80 меняется не сильно, то тупо как 100x80+(x-100)x15=35 через сохранение энергии решаешь и всё. Если же считать что там есть какие-то изменения, то ебаться придется еще с интеграллами, но тоже просто вроде, но НИНУЖНО раз ты школьник.
361 422104
>>2103
фикс.
100x80+(x-100)x15=35x(100+x)
гриб.jpg360 Кб, 2019x2019
362 422121
Поясните за квантово-волновой дуализм. На ионы, молекулы и т.д., на макрообъекты, это распространяется? Т.е. получеатся, что Ленин гриб и одновременно радиоволна макрообъекты можно рассматривать как с дискретной, так и с непрерывной точки зрения?
363 422122
>>2121
Берёшь интерференцию фуллёренов-С60 и изучаешь...
15068445161570[1].gif3,7 Мб, 360x270
364 422125
>>2076

>камеру не забудь взять, чтобы пруфы завезти


бля...
366 422129
>>2104

>100x80+(x-100)x15=35x(100+x)


Идею формулы вроде понял.

>через сохранение энергии решаешь и всё.


Скорее всего оно, только сейчас осознал, что задачу можно упростить, до "Сколько нужно в кипяток долить холодной воды, что бы температура опустилась до необходимой"

А вот при попытке узнать результат вышла проблема. Если для исходных значений калькулятор выдал ответ в 150 литров (пик 1), что хоть и кажется слегка заниженным, но вполне реалистично, то на моем реальном домашнем бойлере вольфрамовский калькулятор сломался (пик 2).

Давай прикинем еще раз, а после я могу попробовать провести эксперимент на практике.
367 422131
>>2129

>100x80+(x-100)x15=35x(100+x)


>Идею формулы вроде понял.


>формулы


>вольфрамовский калькулятор сломался



У меня от этого - "Чувство силы"
http://lib.ru/FOUNDATION/feelpowr.txt

(Это не говоря, про то, что обе формулы неправильные.)
368 422132
>>2129
там идея в формуле правильная, но реализация чет не очень. Всё же если через закон сохранения писать, то должно быть так.
(80-35)x100=(35-15)*x; где x это сколько воды надо долить. и тогда x выходит 225 литров. а из всего бойлера получится 325 литров воды 35 градусов.
369 422134
В журналах РЕН-ТВ стиля встречается понятие поехавших лжеученых "полевые формы жизни". А может ли наука однозначно это опровергнуть? То есть абсолютно невозможно даже если ты Боженька сформировать такую конфигурацию электромагнитного или любого другого поля, которая будет иметь некоторые свойства жизни хотя бы размножение: была одна " структура", потом их стало две?
370 422135
>>2037
тебе в /spc/, в телескопотред
371 422137
>>2131

>У меня от этого - "Чувство силы"


Совершенно не умею в числа. И не имею ничего против людей, которые могут считать в голове если это им легко дается. А вот там, где практика присутствует мне намного легче - в технику и электронику вполне могу. Так что, каждому - свое.

>>2132

>а из всего бойлера получится 325 литров воды 35 градусов.


Вот эта оценка уже по мне, хотя тут наоборот кажется преувеличенной.

Но для моего домашнего бойлера выходит, что я смогу получить 103,5 литров.
И тут есть некоторое несоответствие - когда пытаюсь набрать ванну, то горячей воды мне всегда не хватает, но это скорее всего, что я с самого начала разбавляю открывая холодную в смесителе. Попробую завтра проверить на практике.
372 422139
>>2134

>однозначно это опровергнуть


Научный метод работает как раз наоборот, нам не нужно ничего опровергать, как раз наоборот - живет себе какая-то нех в виде эм-поля и замечательно - надо приручить и использовать вместо радиосвязи.
373 422140
>>2134

>хотя бы размножение


Но ведь поле размножается: подносишь магнит к железу, получаешь два магнита...

>понятие поехавших лжеученых


В науке, делаешь утверждение - доказываешь его. Почти как с пацанами в падике.
Вот сейчас ты должен будешь ответить очком и за поехавших, и за лжеучёных.
374 422144
Насколько сильно был бы полезен в науке и хозяйстве кусок вещества, неплавящийся ни при какой температуре? Пусть это будет нековкий материал ибо слишком чит, чугуний тот же. Каково будет наилучшее ему применение?
375 422145
>>2144
В переносной холодильник его положу. Это ведь сухой лёд, верно? Не плавится, сразу испаряется.
376 422146
>>0966
Боже, я с этим вопросом уже года два доебываю знакомых физиков. Вот уж никогда бы не подумал что ответ можно найти на дваче...
377 422147
>>2146
У Фейнмана хорошо рассмотрено. http://ilib.mccme.ru/djvu/bib-kvant/ked.htm
Тут для гуманитарев, но прям очень как надо. Короче он красавчик.
378 422148
>>2145
Не испаряется тоже, ну чё ты сразу((
379 422155
>>0851 (OP)
Вопрос про энтропию, нет, я не тот идиот с десятком отдельных тредов по 4 поста каждый.

Берем стакан, наполовину с песком/наполовину с водой. Энтропия мала - песок отдельно от воды снизу, да и холоднее на пару десятых градуса.

Начнем вносить энергию в стакан путем взбалтывания ложкой - энтропия повысится, песок равномерно перемешается с водой, да и температура распределится равномерно в равномерной взвеси.

Перестанем вносить энергию (уберем ложку) - энтропия опять понизится - песок осядет на дно, образуя границу с водой.

Ну вы понели, противоречие получилось.
380 422156
>>2155
Нет. так как ты неправильно прочитал второй закон. Он типа про изолированные системы. Но если ты рассмотришь стакан и комнату в которой греешь. С генератором внутри комнаты, и комната изолированна, то там всё топчик. Дело в том, что когда ты перестаешь греть, энтропия стакана уменьшается, но этот стакан отдает энергию во внешнюю среду, а она греется и увеличивает свою энтропию, в итоге в сумме выходит больше нуля.
381 422157
>>2155
Ты, когда взбалтывал - был частью системы и в процессе расцеплял глюкозу/АТФ в своих мозгах мышцах - вот это все тоже нужно учитывать.
cellresp.jpg56 Кб, 400x241
382 422158
>>2157

>расцеплял


расщеплял
383 422159
>>2156
>>2157
Спасибо
384 422161
>>2140

> Вот сейчас ты должен будешь ответить очком и за поехавших, и за лжеучёных.


Лол, давай базарь говно, раз начал тут фуфло гнать пидор.
385 422162
Если мне не изменяет склероз, на планете одноклеточные жили около миллиарда лет, прежде чем появились многоклеточные. А есть ли литература по переходному звену, то есть по простейшему двухклеточному или трехклеточному огранизму? Как трехклеточный организм вообще размножаться будет, продольно поперечным сечением как сейчас червей дождевых лопатой размножают?
386 422163
Дана масса радиоактивного вещества и его период полураспада. Как вычислить время полного распада вещества ? Со школы этого не понимал, ведь не такой степени, чтобы число при возведении в неё было равно 0. А в школе проблем не было ведь, вот честное слово, ни одной задачи с полным распадом всего вещества не было. Были только типа за тоже время что распалось столько то вещества 1 распалось столько то вещества 2 и нужно что-нибудь вычислить. И вообще вот как понимать период полураспада: осталась 1 или нечётное кол-во молекул что с ней делать, она не может больше полу распасться ?
387 422165
>>2162
Короче был какой-то вид клеток. которым было выгоднее жить колонией. Они не были одним организмом, но жили стаей. Попозже. выяснилось. что еще удобнее будет срастись вообще, а потом разделить свои функции. Но литра об этом тооочно есть.
388 422166
>>2163
Ой, ой, ошибся.

>осталась 1 или нечётное кол-во молекул


Атом, конечно не молекула. Атом.
389 422168
>>2163

>Как вычислить время полного распада вещества ?


- такого понятия не бывает.

>честное слово, ни одной задачи с полным распадом всего вещества не было


- что говорит о вменяемости составителей задач.
В практическом плане, можно говорить:
"По истечении такого то времени, вероятность обнаружить столько то атомов данного ве-а, при таком стартовом количестве - такая то."

Можно вычислить время, за которое N-ное количество - распадется, скажем на 98%.
Например - если стартовое количество 1(один) атом, то по прошествии некого времени t - вероятность его распада - 0,98.
390 422170
>>2163
Короче. Радиоктивное ядро. имеет вероятность распасться. И она постоянна. Как ты понимаешь, один атом. может распасться сразу, а может пролежать в три раза дольше, это понятно. Но в любом случае чем больше времени проходит, тем вероятнее, что за ВСЁ это время он распадется. А теперь возьмем сто атомов. Ну если вероятность распасться за секунду у атома 5%. То через секунду, из 100, останется где-то 98-94, может все 100. И эти веротяности можно обсчитать, вспомни школьный теорвер. А теперь мы берем блять 60200000000000000000000 атомов, ты же понимаешь что из такого огромного числа, там будет распадаться оооочень близкое к 5% количество. Пока количество не упадет до тысяч, можно тупо приближать как 5% от всех. ну даже для миллиона, выходит, что вероятность того, что распадется k из миллиона, будет сочетания из k по миллиону умножить на 0.05^k*0.95^(1000000-k); если ты построишь такие вероятности, то для 50000 и около того, будет очень такой хороший максимум. Это биномальное распределение как оно есть. В принципе, его можно неплохо приближать распределением Пуассона. А среднеквадратичное отклонение будет 218 атомов. Ну тоесть колебания от 50000 почти всегда будут укладываться в 49...51 тысяча. А это для миллиона, У нас миллиарды миллиардов их там. Вот и смотри)
391 422171
>>2170
Ок, спасибо. Теорвера у меня в школе не было и я не уверен, что вообще его правильно понимаю. В общих чертах твой пост звучит убедительно для меня, я попробую почитать в этом направлении что-нибудь.
392 422172
>>2165
А, понел. Как медузы получается?
393 422174
>>2170
Раз уж зашла речь про теорвер. Допустим у нас есть малое количество распадающегося вещества(сравнительно не много атомов,скажем 10 в 6 степени), но видимое в очень мощный микроскоп. Получается может быть так, что мы на него смотрим, и раз, всё вещество исчезло, т. к. распалось. Только это очень мало вероятно, но возможно.
394 422179
Ученые, почему вкусы фарша и стейка из одного и того же куска мяса (без специй, кроме равного количества соли и перца) различаются?
395 422182
>>2179

> почему


размер волокон
396 422183
>>2182
Размер имеет значение для вкусовых рецепторов?
397 422184
>>2174
да, лол.
398 422187
>>1247
Не все детерминированные процессы вычислимы. Даже так скажу: большая часть логически непротиворечивых систем невычислимы.
399 422188
>>1221

> Где пруф, что это состояние не было определено изначально?


В неравенствах Белла
400 422191
>>0851 (OP)
Первый, `хитрый` вопрос для всех граждан России, планеты: прошу составить график изменения температуры Вселенной – от первой секунды существования до сегодняшнего дня!? Назовите точную цифру температуры в конце каждого миллиарда лет! Прошу назвать цифры последующих 14 млрд лет, а особо рискованных прошу назвать и конечную цифру температуры вселенной. Этот вопрос должен выявить нескольких настоящих ученых – им мы и доверим наше будущее. Вопрос второй: прошу вычислить размеры Вселенной, основываясь на данных, расчетах любимого и почитаемого всеми Эйнштейна, с поправками суперактивного, плодовитого Хоккинга!? Ученые планеты сообщают нам, что они знают скорость расширения Вселенной, знают, что расширение замедлялось, а сейчас ускоряется!…и знают, как и мы, что расширяется наша Вселенная уже 13,7 млрд лет. Осталось только умножить эти две цифры – скорость и длительность расширения… Сам понимаю, что школьники младших классов решат эту задачу намного быстрее чем известные академики…потому что им нечего скрывать и для них важно только правда, истина! У всех Вас получится очень небольшая Вселенная…но это по Эйнштейну. Вопрос третий: все галактики должны лететь из одного центра, от Большого взрыва…а летят как попало, еще и ускоряются почему-то (я лично знаю почему – все они уже проделали половину пути, а вторая часть пути – это подлёт (притяжение) к встречной черной дыре!)!? Вопрос: тогда зачем и как часть галактик поменяли свои направления? Вопрос четвертый: все люди на планете своими будильниками и часами… секундами измеряют: время концерта, войны, жизни!…Но секунда не принадлежит времени!…секунда – наша родная земная мера, которая измеряет длительности процессов!…длительность концерта, если это нам надо, длительность, а не время концерта!!! Время – слово-паразит, время не существует, а есть процессы и их длительность!…еще есть шкала (24 часа), на (по) которую мы планируем свои мероприятия! Вопрос: кто первый разместил в «справочнике для школьников» такую информацию: «Время мы измеряем секундой»? И еще вопрос: кого беспокоит эта фальшивая информация, кто отвечает за образование детей и будущее планеты? Чем грозит невежество выпускников школ, бесконтрольная работа ученых, доверчивость наших детей? А виноваты – заинтересованные ученые или наша апатия? Вопрос пятый: откуда произошло слово «относительность», имеет ли оно какие-то параметры, значения, как и где его можно заметить? Когда-то Галилей колдовал над «принципом относительности», над которым работали и многие из его предшественников!? Все мы понимаем слово «относительность» от слова «отношение», но даже слово «отношение» чаще бывает совершенно пустым, ненужным словом: два боксера выясняют отношения…нет, они дерутся, соревнуются! Муж и жена выясняют отношения…нет, если точнее: они ругаются и дерутся. Вопрос: если Эйнштейн наделил разным временем коров в стаде, пчел, мух, летящие ракеты, людей, которые летят на этой ракете и для которых несуществующее время бежит с разной скоростью, то чем он и его последователи измеряют относительность этих мух, которые свободно летают…чем, кроме времени, которого нигде нет? Вопрос шестой: реликтовое излучение от нашего Большого взрыва, которое якобы поймали наши ученые, давно, еще 13 млрд лет назад улетело вперед, обогнав нас!? Вопрос: чье же реликтовое излучение поймали наши ученые и с такой любовью и пониманием изучают его!?? Вопрос седьмой: если мы докажем, что Вселенная не расширяется, то получается что бозон Хиггса больше не нужно искать!?…не нужно искать и струны, и кротовые норы и искажения пространства? Сэкономим народам планеты триллионы долларов! Или будем искать вместе с ними?
400 422191
>>0851 (OP)
Первый, `хитрый` вопрос для всех граждан России, планеты: прошу составить график изменения температуры Вселенной – от первой секунды существования до сегодняшнего дня!? Назовите точную цифру температуры в конце каждого миллиарда лет! Прошу назвать цифры последующих 14 млрд лет, а особо рискованных прошу назвать и конечную цифру температуры вселенной. Этот вопрос должен выявить нескольких настоящих ученых – им мы и доверим наше будущее. Вопрос второй: прошу вычислить размеры Вселенной, основываясь на данных, расчетах любимого и почитаемого всеми Эйнштейна, с поправками суперактивного, плодовитого Хоккинга!? Ученые планеты сообщают нам, что они знают скорость расширения Вселенной, знают, что расширение замедлялось, а сейчас ускоряется!…и знают, как и мы, что расширяется наша Вселенная уже 13,7 млрд лет. Осталось только умножить эти две цифры – скорость и длительность расширения… Сам понимаю, что школьники младших классов решат эту задачу намного быстрее чем известные академики…потому что им нечего скрывать и для них важно только правда, истина! У всех Вас получится очень небольшая Вселенная…но это по Эйнштейну. Вопрос третий: все галактики должны лететь из одного центра, от Большого взрыва…а летят как попало, еще и ускоряются почему-то (я лично знаю почему – все они уже проделали половину пути, а вторая часть пути – это подлёт (притяжение) к встречной черной дыре!)!? Вопрос: тогда зачем и как часть галактик поменяли свои направления? Вопрос четвертый: все люди на планете своими будильниками и часами… секундами измеряют: время концерта, войны, жизни!…Но секунда не принадлежит времени!…секунда – наша родная земная мера, которая измеряет длительности процессов!…длительность концерта, если это нам надо, длительность, а не время концерта!!! Время – слово-паразит, время не существует, а есть процессы и их длительность!…еще есть шкала (24 часа), на (по) которую мы планируем свои мероприятия! Вопрос: кто первый разместил в «справочнике для школьников» такую информацию: «Время мы измеряем секундой»? И еще вопрос: кого беспокоит эта фальшивая информация, кто отвечает за образование детей и будущее планеты? Чем грозит невежество выпускников школ, бесконтрольная работа ученых, доверчивость наших детей? А виноваты – заинтересованные ученые или наша апатия? Вопрос пятый: откуда произошло слово «относительность», имеет ли оно какие-то параметры, значения, как и где его можно заметить? Когда-то Галилей колдовал над «принципом относительности», над которым работали и многие из его предшественников!? Все мы понимаем слово «относительность» от слова «отношение», но даже слово «отношение» чаще бывает совершенно пустым, ненужным словом: два боксера выясняют отношения…нет, они дерутся, соревнуются! Муж и жена выясняют отношения…нет, если точнее: они ругаются и дерутся. Вопрос: если Эйнштейн наделил разным временем коров в стаде, пчел, мух, летящие ракеты, людей, которые летят на этой ракете и для которых несуществующее время бежит с разной скоростью, то чем он и его последователи измеряют относительность этих мух, которые свободно летают…чем, кроме времени, которого нигде нет? Вопрос шестой: реликтовое излучение от нашего Большого взрыва, которое якобы поймали наши ученые, давно, еще 13 млрд лет назад улетело вперед, обогнав нас!? Вопрос: чье же реликтовое излучение поймали наши ученые и с такой любовью и пониманием изучают его!?? Вопрос седьмой: если мы докажем, что Вселенная не расширяется, то получается что бозон Хиггса больше не нужно искать!?…не нужно искать и струны, и кротовые норы и искажения пространства? Сэкономим народам планеты триллионы долларов! Или будем искать вместе с ними?
401 422192
>>2183
Вкусовые рецепторы реагируют на пептиды в основном, а их количество при поджаривании зависит от размера волокон.
402 422193
>>2191
Ебать хуйни понаписал
403 422194
>>2193
Это скорее всего комментарий с ютуба. Там частенько такая дичь проскакивает.
404 422195
>>1228

>Почему не могут решить проблему трех тел, те вывести формулу, по которой можно точно узнать,


>где будут тела через х времени?


Вот тут вроде как нашли ещё несколько частных решений этой задачи: https://lenta.ru/news/2017/05/15/152/

>есть строгая же зависимость: начальные параметры>время>текущее положение.


>>1248

>Зависимость есть, а формулы нет, прикинь. Ждет когда ты ее выведешь.


Там же расклад сил динамический.
>>2187
Мне кажется что ты, перефразируя Теорему Гёделя о неполноте,
забыл добавить уточнение о том,
что непротиворечивые система невычислима
средствами этой самой формальной системы,
если рассматривать эту формальную систему как арифметику,
т. е. если её функциональные и структурные элементы
способны производить арифметические операции и выводить всякие форулы и уравнения.

А вообще, движение системы трёх тел, можно достаточно точно смоделировать и вычислить,
если упрощённо рассмотреть систему как одно тело, но побольше. Ведь центр масс у системы не меняется.
406 422197
>>2193

>Я не понял вопрос или я не могу на него ответить


>Скажу, что вопрос хуйня


Что на наукаче забыл, тем более в этом треде?
407 422198
>>2197
Ты хоть видел сколько там вопросов в этом полотне?
Хоть бы сброс строки пришпандорили.
408 422199
>>2196
В комментах под научпоп-видео и не такое встретишь.
409 422200
>>2197
Я всё понимаю, там половина вопросов сформулирована через жопу, а другая половина в себе же содержит фактические ошибки.
Как связаны расширение Вселенной и Хиггс, например? Одному автору сего опуса известно
410 422201
>>2147
Спасибо, Анон
412 422210
>>0851 (OP)
Наплыв плоскоземельщиков в последнее время - это такой глобальный троллинг тупостью или мировая деградация разума у людей?
413 422211
>>2210
Троллинг. Но этот троллинг не понимают реальные плоскоземельщики, и поэтому тоже повылазили, мол "ну нас много".
414 422213
>>2064

>лунный диск идентичен солнечному


не совсем

>вращение и оборот вокруг Солнца никак этого не меняют


меняют в течение года
415 422214
>>2210
Еще один дурень думающий что она круглая.
bitch.jpg154 Кб, 1920x1080
416 422215
Поясните за Расширение Вселенной.

Галактики движутся в некоторых направлениях с некоторой скоростью от гипотетического центра Вселенной (эта скорость, вероятно, примерно одинакова у всех).

Кроме того, пространство расширяется в каждой точке с некоторой скоростью.

И как пересекаются двех этих скорости, и что будет когда их сумма превысит скорость света?
417 422218
>>2214
Ну нинад.
418 422219
>>2215
Центра нет, даже гипотетического.
Галактики из-за гравитациинаоборот сближаются, а пространство растягивается, да.
Сложение скоростей так не работает.
419 422221
>>2219
Т.е. галлактики не получили от Большго Взрыва никакого импульса и не имеют собстенной скорости движения?
420 422222
>>2219
Ты не можешь утверждать, что сложение скоростей в данном случае невозможно, т.к. не сможешь это доказать, в принципе.
421 422223
>>2221
они все имеют свои скорости. Но большой взрыв это больше именно к пространству.
422 422224
>>2223
Так их собственные импульсы разве не помешают им никогда не сойтись под действием гравитации, при условии, что они находятся на большом расстоянии, даже если не было бы расширения Вселенной?
423 422225
>>2224
Так взрыв то был не из точки, а во всех точках пространства
424 422227
>>2225
Серьёзно? Просто точка в реальности сущестовать и не может...

Вопрос в том, собарлись бы все галлактики в одну когда-нибудь или нет? Или гравитация была бы преодолена и большинство из них так бы бесконечно и разлетались бы? (мы не учитываем тепловую смерть вселенной итд)
425 422228
>>2227
и это без учёта расширения пространства
426 422230
>>2228
Слиплись бы, так как они слабо двигаются относильно пространства, ну ты сам подумай. Если везде всё "бахнуло". то как выделить центр, от которого всё разлетается? НИКАК
427 422232
>>0851 (OP)
Кароч, препод предлажил мне вкатится в симулирование сетей ns3. Один из асперов сказал что за эту хуйню компанииготовы полатить больште денги. Но правда ли это? Сложно вкатываться в эту штуку, не хочется впустую потратить время и жизнь.
428 422233
>>2230
Почему они слабо двигаются? Я вот думаю они двигаются с огромной скоростью.
429 422234
>>2230
По формуле, навереное, расчитать, вот как.
bitch.jpg154 Кб, 1920x1080
430 422235
А как насчёт, нет никакого расширения Вселенной, а есть неизученные эффекты света, которые видны только на огромных расстояниях?
431 422236
>>2234
>>2233
По формуле как раз они слабо двигаются, ибо центра взрыва нет, и СТО и ОТО не выделяет какие-то направления в пространстве.
432 422237
>>2235
Теорию "старения" света гугли. Её проверили и опровергли.
433 422239
>>2235
ускоренное расширение видно по 6 независимым эффектам
434 422243
>>2233

>Я вот думаю они двигаются с огромной скоростью.


Расстояния огромнее на несколько порядков.
435 422244
>>2235
Ты в эфир наверное веруешь?
436 422246
Почему световая скорость замедляет время? Не догоняю.
437 422247
>>2246
Потому что пространство и время - это единое целое. Нельзя изменить скорость в пространстве, не изменив во времени
438 422248
>>2236
что ты несёшь вообще?
439 422249
>>2237
Но он рассеивается.
440 422250
>>2239
подробнее
441 422251
>>2246
для фотона время вообще не идёт и мы его наблюдаем, как нечто замороженное.

Точнее так было бы, если бы СТО была правдой.
442 422252
>>2247

>Нельзя изменить скорость в пространстве, не изменив во времени


А почему замедляет? Ускорять же по идее должно. Или такая скорость как-то искривляет пространство?
cosmonavt.png1,5 Мб, 1920x816
443 422253
А вы в курсе, что при расширении пространства, по СТО расширяется пространство-время, а следовательно и время.

Время становится длиннее, растянутее и все процессы идут всё медленнее и медленнее, относительно того, что было раньше.

Мало того, чем обеъекты удалённее, тем медлнее в них течёт время относительно друг друга и когда-нибудь t может стать = 0. Такие дела.
444 422254
>>2252
Грубо говоря, у тебя всегда постоянная скорость в 4х мерном пространстве-времени. Когда ты стоишь на месте, вся скорость уходит на течение времени. Когда летишь с околосветовой скоростью, течение времени должно соответственно замедляться
Вот тут хорошо это показано, и ещё есть прога в конце, чтобы самому поиграться
https://habrahabr.ru/post/169347/
445 422255
>>2254

>Грубо говоря, у тебя всегда постоянная скорость в 4х мерном пространстве-времени.


Из-за расширения вселенной?
446 422256
>>2253
Ебануться.
447 422257
>>2255
Нет, по СТО. Причём тут расширение вселенной?
cosmonavt.png1,5 Мб, 1920x816
448 422259
Проведём прямую линию на плоскости между двумя точками.
Теперь добавим третье измерение и выгнем в нём нашу линию по дуге.
В трёхмерном пространстве линия стала длиннее.
Убрали третье измерение, линия стала короче.

Была линия некоторой длины в трёхмерном пространстве.
Разогнали её до скорости, близкой к скорости света.
Длина линии уменьшилась.

Но куда делась длина? Да ничто и никуда не девалось, оказалось, что пространство четырёхмерно и это (якобы) пространство-время.

Только вот четвёртое измерение должно быть длиной, а не временем, как у Эйнштейна, чтобы длина могла изменяться таким вот образом, вы же не можете изменить длину линии по оси времени, нужно полным идиотом для этого быть.

А время - уже пятое измерение. Но тогда все формулы Эйнштейна не верны, а точнее неполные, т.к. в одних случаях работают, а в других нет. Вот и всё.

Я только что решил все проблемы СТО и ОТО. Слава Моему Величию.
449 422267
>>2259

> Добавил измерение - решил проблему


Иди 26 измерений посчитывай.
Не как что-то плохое
450 422269
Там это... Нобелевку вручают. Почему нет прикреплённого треда, по такому случаю ?
451 422270
>>2267
Достаточно пяти, шизик.
452 422271
>>2269
потому что мне не вручают
453 422272
>>2252

> Ускорять же по идее должно.


По чьей "идее"? Твоей?
454 422275
Почему при озонировании курицы выделяется пенистая шняга и из чего она состоит?
https://www.youtube.com/watch?v=uSHd_NCYIHU
455 422278
>>2275
кек, просто окислил
456 422279
>>2248
Практически все теории базируются на ОТО. При этом в самом ОТО полнейшая симметрия, и нет выделенных направлений. Пиздец, я конечно понимаю что "большой взрыв" можно понять буквально, но не настолько же.
457 422280
>>2275
Там при разложении озона выделяется атомарный кислород,
а это сильный окислитель, очень активный и он короче окисляет всякие ткани и прочее, с чем соприкасается.
Попробуй полить эту курицу перекисью водорода примерно то же самое увидешь.
При разложении перекиси тоже выделяется атомарный кислород.
458 422281
>>2251
хех, сто это самое первое, что вообще возникло. Надо смотреть на КЭД. Там фотоны рассмотрены. И им похуй на время, как и на само своё положение. Просто возмущение поля.
459 422282
>>2253
Для нас это будет незаметно, ибо если весь мир медленнее, то ничего не медленнее. Привет теория относительности.
460 422283
>>2280

>Там при разложении озона выделяется атомарный кислород,


Ноу щит.
А что конкретно образуется по составу? Окись жира? Окись плазмы крови? Есть у них какие-то названия?
461 422284
Пацаны, нужна помощь. Я заебался уже целых 2 часа искать статью. Я ее проебал когда-то давно. Помню, что в статье было про: 1. влияние оперантного обусловливания на то, согласен человек со информацией или нет. 2. влияние классического обусловливания (вот тут подавали запах) на воспринимаемую информацию. В одном из экспериментов людям говорили рекламный лозунг и они должны были оценить его. В статье также была ссылка на исследование такого рода подкреплений на журнал связанный с чем-то типа экспериментального анализа поведения. 2 часа и нихуя. Я всю перепискую свою перерыл и никак не могу найти эту ебучую статью.
462 422285
>>2253
>>2259
У меня после тебя время сферически растягивается.
Screenshot2017-10-04-10-32-22.png369 Кб, 1280x800
463 422287
Двач помоги с задачей
464 422288
>>0851 (OP)
Как понять распадается моя психика или нет? Если крыша едет, чем её можно подпереть чтобы не ехала?
465 422290
>>2033
Лол.
466 422296
>>2279
Тогда у галактик и собственной скорости движения не было бы.
467 422297
>>2296
Она у них есть, но от того, что они стягиваются в скопления.
468 422298
>>2297
Ага и при этом у всех одинаковая скорость.
469 422299
>>2297
У Земли, Луны и Солнца скорость есть потому, что ни стягиваются в скопление.
470 422300
>>2283
закон сохранения энергии тебе в руки
471 422301
>>2298
Блять, бесите. нахуй вас реально. Пытаешься что-то объяснить, так начинают вместо нормальных вопросов нести сука хуйню. Идите блять сами читайте литературу. Не забудьте сначала теорию поля почитать. Долбоебы блять, сами спросили какая скорость у галактик если не учитывать скорость от расширения. Отвечаю что в среднем 0. Так хуйню стали нести.
sc.jpg154 Кб, 1920x1080
472 422302
Почему вы все думаете, что это пространство расширяется, а не мы все уменьшаемся?
473 422303
>>2302
Лол.... вопрос семантики же.
aeb5f44099c1cdb160ea9df21b01211b (1).png26 Кб, 160x160
474 422304
>>0851 (OP)
Поясните за металлургию в космосе (в вакууме и с микрогравитацией).
Что там такого можно сделать, чего не сделать на земле? Можно ли получить какую-нибудь здоровенную условную бабку в монокристаллическом виде с прочностью в 5 раз больше чем на земле?
475 422308
А правда, что ученые до сих пор не могут объяснить почему велосипед едет и не падает?
477 422310
>>2308
Нет. Но вот объяснить, как шмель не нарушает законы аэродинамики, пока никто не может.
478 422311
>>2310
лол, я думал уже никто не верит в эту байку.
479 422313
>>2304
1) Примесей нет, ибо вакуум;
2) Гравитация не мешает образовывать идеальную кристаллическую решетку.
480 422315
>>2311
Это не байка, это наука, друг. Если когда-нибудь учёные узнают, как летают насекомые, то можно будет создавать очень маленькие и компактные летательные аппараты. Судя по тому, что даже инженеры не могут изготовить таких компактные летательные аппараты, то вывод очевиден. Как летают насекомые - загадка.
481 422316
>>2304
ничего нового. в некоторых местах космоса весьма, скажем так, интересное давление, температура, элементы и др. находясь в этих условиях можно соорудить неведомую ебанину, но она будет таковой в этих условиях. в других условиях ебанина развалится или схлопнется или закипит или замерзнет или еще чего т.е. особой пользы нет. да и мы до того крутые ребята, что чем лететь куда-то, проще создать те самые интересные условия на земле
482 422317
>>2315
Если когда-нибудь учёные узнают, как сокращаются анусы, то можно будет создавать очень маленькие и компактные протезы анусов. Судя по тому, что даже инженеры не могут изготовить таких компактные протезы анусов, то вывод очевиден. Как сжимаются анусы - загадка.
Шутка.webm1,6 Мб, webm,
540x540, 0:16
483 422318
>>2315
Байка про шмеля который не подчиняется законом аэродинамики, пошла после того, как к нему применили формулу, которую вывели для расчета обычных самолетов пассажирских. Эта формула скорее всего и для истребителей даст хуйню. В этой формуле блять даже не учитывалось, что крылья двигаются. О чём ты. Полет шмеля не такая сложная задача, чтобы "ой ой мы не знаем как объяснить". Всё знают, проблема не в "как?" проблема в "материалы дайте"
484 422319
>>2318
Ага. Формула доказывает, что шмель с его крыльями планировать не может. Спойлер: истребители тоже.
485 422320
>>2319
Ну, так и есть. Шмель если крылышками перестанет махать, планировать не будет. Смотри ка. Формула работает. А люди любят красивые байки таскать.
486 422321
>>2317
Аналогия - не агрумент, креационист.
Сокращение сфинктера уже давно объяснено учёными.
487 422322
>>2321
Как и полет насекомых)
488 422323
>>2318

>после того, как к нему применили формулу, которую вывели для расчета обычных самолетов пассажирских.


Нет. К нему применили научную формулу аэродинамики. Расчёты подтверждены учёными и инженерами. Если хочешь, могу найти их фамилии.

>Всё знают, проблема не в "как?" проблема в "материалы дайте"


Какие материалы нужны для концепта мелкого летательного аппарата?
489 422324
>>2322
Самолёт делали, основываясь на аэродинамики птиц. Была создана теория => был создан концепт => теория подтвердилась => началось промышленное внедрение самолётов.
На какой стадии находится разработка летательных аппаратов на основе аэродинамики насекомых?
490 422325
>>2324
ЛА нет. Физика не позволяет, про закон квадрата-куба слышал? А летающие кибершмели есть.
491 422326
>>2323
научная формула аэродинамики.
Кек блять.
492 422327
>>2315

>то можно будет создавать очень маленькие и компактные летательные аппараты


Цесна, GEN H-4, Mosquito Air, квадрокоптеры дохуя большие?
image.png258 Кб, 450x535
493 422328
>>2325

>закон квадрата-куба


Противоречит наблюдениям. По этому закону в плане борьбы азиатский карлик будет сильнее, чем здоровенный негр-качок.
Более того, этот закон очень сильно устарел. Он говорил о том, что пауки больших размеров невозможны, поскольку это вызвало бы их удушье, но после открытия Австралии и островного гигантизма, мы видим, что австралийские пауки в несколько раз больше того же вида на континенте.
494 422329
>>2328
Ты, похоже, просто не знаешь его сути. И таки да, два раскачанных карлика той же суммарной массы раскатают негра.
0VhThALT88o.jpg57 Кб, 546x604
495 422331
>>2329

>И таки да, два раскачанных карлика той же суммарной массы раскатают негра

image.png129 Кб, 966x871
496 422332
497 422333
>>2304

> Поясните за металлургию в космосе (в вакууме и с микрогравитацией).


хуита, металлы активно испаряются и всё идёт по пизде
498 422336
>>2313
Профит-то большой будет? Можно строить прочные йоба-ssto?

>>2316
Как ты микрогравитацию создавать будешь?

>>2333
Техническая проблема.
image.png42 Кб, 160x160
499 422337
Правильно ли, что эволюция идёт по такому принципу: трава-куст-дерево?
Можно ли селекцией или генной инженерией превратить куст в дерево или куст в траву?
500 422338
>>2337
нет, не правильно. переход из одного вида в другой невозможен в естественной среде и любой, кто будет доказывать обратное-требуй пруфов, которых нет.
вообще дарвин обосрался, можно подробней обсудить, но мне сейчас немного лень.
селекцией? нуууууу можно очень сильно озадачится и наделить, например, расстение ртом (озадачится потому, что надо чтоб носитель его не отторгал, а тот гармонично вписался в быт расстения), лапками и др. признаками, не присущих этому виду. ну окей, допустим, получилось. это лишь доказывает, что без вмешательства из вне это невозможно.
502 422341
>>2337
Ну, смотря что называть деревом, банан это трава, например.
Превратить можно. Вишня вообще на переходной стадии.
503 422346
>>2336

> Можно строить прочные йоба-ssto?


бесполезная сайфайная ебанина

> Как ты микрогравитацию создавать будешь?


она везде есть

> Техническая проблема.


решений пока нет
504 422349
>>2346

>бесполезная сайфайная ебанина


Хочу проводить выходные на сатурне.

>она везде есть


Че?
505 422350
>>2349

> Че?


через плечо

> Хочу проводить выходные на сатурне.


а причем тут фентазийное ссто?
откуда вы его берёте?
506 422354
>>2350

>через плечо


Поясняй где ты микрогравитацию взял на земле. Вроде бы она одно из главных условий для формирование монокристаллов. Хотя я в этом не уверен.

>а причем тут фентазийное ссто?


Ну епта, повышаешь прочность лопаток турбины = можно за сто рейсов не перебирать лопатки. Ну и прочая такая хуйня вот.
Может там можно выращивать идеальный кристалл кварца, шобы прям держал температуру газофазного урания, ну ты понел)))0000

>откуда вы его берёте?


Аниме, очевидно же, всё оно виновато.
507 422355
>>2354

> Поясняй где ты микрогравитацию взял на земле.


везде в космосе, вроде же про космос речь идёт, не?
да и чистота не магией получается материал изначально должен быть очень чистым, вакуум нужен чтобы ещё больше при обработке не засрать, для начала надо ещё выгородить чистый производственный цикл до этого момента, опять же металлы в вакууме активно испаряются, теряет в массе, засирая и вакуум и производственные поверхности

> Ну епта, повышаешь прочность лопаток турбины = можно за сто рейсов не перебирать лопатки. Ну и прочая такая хуйня вот.


для этого не нужно ссто от слова совсем

> Аниме, очевидно же, всё оно виновато.


реальный мир суров, в нём ссто только маняпланы с манякартинками
508 422363
>>2355

>да и чистота не магией получается материал изначально должен быть очень чистым


>опять же металлы в вакууме активно испаряются, теряет в массе, засирая и вакуум и производственные поверхности


Как считаешь, возможно какие-то компоненты двигателей или теплозащиты прям в космосе и хуярить? Ну тип изготовить балку номер 9 корпуса корабля как монокристалл. Сейчас говорят про образование усиков, но может пар-то хотя бы в теории будет осаждаться на балке превращая её в монокристалл? Чтобы прям сделать шаттл которому вообще похуй на атмосферу.
Для металокерамики вроде как это вообще топчик будет, без лишних примесей увеличиваются в десять раз характеристики, к тому же её можно напрямую с металлом сплавлять.

>для этого не нужно ссто от слова совсем


Для людей, для людей жи есть. Доступный космос, все дела.

>реальный мир суров, в нём ссто только маняпланы с манякартинками


Так в реальном мире пока что нет металлургии в космосе!
509 422374
>>2363

> Как считаешь, возможно какие-то компоненты двигателей или теплозащиты прям в космосе и хуярить?


возможно конечно же, но на текущий момент это невообразимо дорого, а также технологически и технически трудно и нельзя спрогнозировать будет ли это решено в отдалённой перспективе
перспективы удешевления крайне сомнительные

> Доступный космос, все дела.


ну ты понел
даже всякие илоны маски бурно врывающиеся в рыночек за счет цен и прочих инноваций не лезут в ссто даже в самых смелых маняпланах, которых имеют просто дохуя

> шаттл


умер и никогда ссто не был
510 422385
>>2338
Мамкин креоцианист? или ты просто за современный взгляд?
511 422412
>>2374

>возможно конечно же


Прост я от слова совсем не понимаю производственный цикл. Ну хуе-мое, превращение в пар, осаждение, дальше-то что? Хуй проссышь короче, но мне почему-то кажется что кол-во производственных циклов из-за халявного вакуума снизится в разы.

>перспективы удешевления крайне сомнительные


Ну я щитаю что после первого завода на луне всё будет хорошо и прекрасно. Если BFR и New Glenn не обосруться, то 200+ тонн за рейс на луне будут. Два десятка рейсов и один завод будет стоять.
512 422461
>>2316
Металлический водород же (по предположениям учёных) останется самим собой при переносе в земные условия. Почему бы не существовать и другим стабильным ебанинам?
513 422462
Считается, что все эмоции имеют биологическое обоснование (то есть как бы возникли в результате эволюции). Тогда как объяснить возникновение зависти? Зависть к хозяину большего количества "передаваемых" ресурсов понятна можно спиздить/отобрать/выпросить, тем самым увеличив своё благосостояние и шанс успешно размножиться. А есть ли основа у зависти к обладателю ресурса, который невозможно отнять? физические/умственные/etc способности
inb4: неисправленная ошибка в эмоциональном центре мозгов
514 422470
>>2461
Я на самом деле не понял с какого перепугу они решили. что металический водород стабилен. Хорошо он метастабилен, в это я верю. Но разве барьер перехода не минимальный? Это же блять водород, лол. У него во всех физхим процессах барьеры минимальные, так этот пидор часто еще и туннелирует через них.
515 422471
>>2462
Точно так же, мотивация не отнять, а получить себе.
516 422485
>>2471
Но, например, длинный хуй никак не "получить себе".
517 422503
>>2485
Тогда ты от завести отрежешь ему хуй. И у тебя будет самый длинный.
518 422528
>>2316
Кстати, по поводу нестабильных ебанин. Может ли теоретически быть "нейтронное вещество" в нейтронных звёздах стабильным без высоких давлений?
519 422529
>>2503
Не хватает картинки "Негр с пальцем".
520 422655
Какую тему для исследовательской по химии взять?
521 422658
>>2655
Моделирование механизма какой-нибудь реакции возьми.
522 422675
>>2658
Скучно, и это, в основном, чистая теория, а я в лабе заниматься хочу
523 422681
>>2675
лаба и пробирочки, как по мне ребячество и самая скука. Люди которые "ооо практика практика", очень напоминают мне детей, которые "ооо взрывать взрывать".
524 422701
>>2681
Когда ты можешь поехать делать работу на химфак, то это не просто "пробирочки", а возможность, например, самому синтезировать интересующее тебя соединение, и изучить потом его свойства
525 422704
>>2701
Да, но чувствуешь себя немного обезьяной от такого. Это методы прошлого века. Надо развивать предсказательную химии. Через 100 лет, мы так и будем просто синтезировать и перебирать лекарства, вместо расчета. Если сейчас каждый химик будет как дятел сидеть под тягой.
Тред утонул или удален.
Это копия, сохраненная 18 января 2018 года.

Скачать тред: только с превью, с превью и прикрепленными файлами.
Второй вариант может долго скачиваться. Файлы будут только в живых или недавно утонувших тредах. Подробнее

Если вам полезен архив М.Двача, пожертвуйте на оплату сервера.
« /sci/В начало тредаВеб-версияНастройки
/a//b//mu//s//vg/Все доски